AMA | October 2023

Welcome to the October 2023 Ask Me Anything episode of Mindscape! These monthly excursions are funded by Patreon supporters (who are also the ones asking the questions). We take questions asked by Patreons, whittle them down to a more manageable number -- based primarily on whether I have anything interesting to say about them, not whether the questions themselves are good -- and sometimes group them together if they are about a similar topic. Enjoy!

AMA

Support Mindscape on Patreon.

Joye Colbeck
Could you give a good update on the Mindscape scholarship recipients, which courses they have chosen and how they are progressing? It feels good knowing we are all supporting the thinkers of tomorrow with our donations.

Alex T.
I’ve enjoyed listening to Mindscape for several years. In many of your AMA answers, you seem to “hedge” quite a bit. I’ve grown to appreciate your approach to truth and knowledge and how the precision of language complicates pinning down objective fact. My question is: are there any subjects or statements that you would consider unequivocally true?

Mark Boucher
PRIORITY QUESTION:
Does the block universe theory imply that motion is unreal/illusion , and sometimes also implied that every moment is a kind of eternity ( not of duration but some other kind which I couldn't understand) and that nothing happens in it , if yes how does the things we observe everyday makes sense and to even have an illusion some kind of motion (neurons firing) seems to be necessary and what has it to do with eternity .

Peter M Caruso
If we could look at our universe from the outside, what would its shape be?

Carlos Nunez
On the topic of misinformation, do you agree with the idea that "freedom of speech doesn't equal freedom of reach"? What would you say to the freedom of speech fundamentalists that believe that any infringement of freedom is negative, even when it prevents the spread of misinformation?

Blake Brasher (like Frasier)
I'm an electrical engineer by trade and I've often thought it is a terrible historical mistake that the charge of the electron was decided to be negative instead of positive. My question is, do you think electromagnetism and electrical engineering would be easier for people to comprehend if we all agreed to swap polarities on all things carrying electric charge, such that anything positive would be negative going forward and vice versa? Or, is there some fundamental reason the charge of the electron should be negative and not positive that I'm simply not understanding?

Michael Fryd
I was under the impression that when a lot of matter occupies a very small space, a black hole will form.
When I hear about the "big bang" it sounds like all the matter in the universe was compressed into a very small space.
I was under the impression, that under our current understanding of the laws of physics, that much matter in such a small space must create a black hole. How was all the matter and energy in the Universe able to escape such a small space preventing a black hole from forming?

Ryan Santos
I feel like the popular speculations about extra terrestrial life are too constrained. Do you think there is any special reason to presume that life elsewhere is necessarily planet bound and subject to restraints like ours?

Tim Gianitsos
There's a famous paper by John Wheeler where he makes some bold statements and argues that they are basically irrefutable. What is the stance of the modern physics community on them?
- the world can't just be a machine ruled by continuum physical law
- there is no such thing at the microscopic level as spacetime
- bits are the most fundamental reality

George Hampton
What lead you to write these new books? Did the pandemic have much to do with it?

Fran Pla
Episode 41'steven strogatz you mentioned you werre taking melatonin when you are in different time zones so to connect it to episode 251 with jet lag eating early and daylight as good measures to adjust our internal clocks so my question is are you still taking melatonin to combat jet lag ?

Stevie CPW
If I read the report correctly, 30 billion suns could fit into a recently discovered black hole. How does a black hole get to be that big, and when you learn of such discoveries do you have any emotional reaction to their awesomeness or is it more matter of fact to you?

Geddy Lee Smolin
As a good Bayesian; how do you differentiate "belief", "hope", and "faith"?

J C
so google is shutting down the podcast application and asking us to listen to podcasts in youtube . i dont like youtube for podcasts and i understand that you podcasters receive less credits for youtube videos than for actual podcasts. where do you suggest we go now to listen to you? thanks!

Ari Moudi
Is it possible that at the very center of a black hole is the same thing as the universe before the big bang?

Anthony Nault
What’s the difference between Wavefunction Realism (of the kind Albert explores) vs Mad-Dog Everettianism?

onlynormalperson
priority question
In an earlier AMA you stated that in contrast to suicide, euthanasia may be allowable, which would indicate you think there should be a general moral and/or legal prohibition on suicide.
At 30 I've come to the conclusion that 1. I'm generally unhappy due to depression, rather than external variables 2. I am completely skeptical that I make people around me happier than if I were to not exist. There are currently living people who would be saddened by my death, but on some level I ultimately hope that I can find an opportunity to check out early, maybe within 10 years, 15 years at latest, without anyone feeling harmed, which will require certain behavioral commitments, but at least seems like an objectively non-futile goal, possibly the only one that is possible in a world where we have petty limited agency to make things better for others, and the second law of thermodynamics implies existence will always be more destructive than constructive.
Do you think creating and working towards harm-mitigated suicide as a long term plan is ethically and logically sound, or would you posit some objection?

*****************************************
robert granese
Priority Question
Is there any credence to the idea that dark matter is space that exists at a level of hgher dimensionality, so we cannot see or interact with it?

Lars Wallaart
we say there are 4 fundamental forces although Einstein showed us that gravitation is merely a distortion of spacetime. Could the other three forces also be considered to be “just” distortions of dimensions? And might these dimensions be correlated to the “extra” dimensions referred to in string theory (curled-up dimensions)?
*********************************************

Brian Mapes
Do you have more specific ideas or examples about how to usefully tally macro-scale entropy (complexity) separately from the abyss of molecular entropy (which increases by the 2nd law)? Inspired by the coffee figure in The Big Picture, and your Berkeley colloquium last year, I am trying to do this for mesoscale patterns or configurations of clouds in weather scenes, which I hope can be a natural framework for evolutionary theory of why naively-"unlikely" but energy-efficient multicellular storms are so common.

Liam McCarty
Why is there so little scatter in the baryonic Tully Fischer relation? I saw you say in a recent interview with Brian Greene that there’s overwhelming evidence for dark matter, but I’ve struggled to understand why there would be so little scatter in the dark matter paradigm. And I’ve been struck by the fact that MOND predicts an exact relation that matches the data. I know MOND fails for galaxy cluster dynamics and more, but I’m curious to hear your thoughts on why it predicts as much as it does, despite being incorrect.

Henry Jacobs
In your recent interview with Brendan Nyhan, he expressed skepticism about the impact of filter bubbles. Did this alter your perception on the issue?

Jason Ricciardi
In your Podcast with Yejin Choi you stated, "When people say alignment in the context of AI, they mean aligning the values of human beings with the values of AI, which sounds like a good thing to do but then again I'm not sure that AI has values. I worry that there's a category mistake going on here." Then Choi responded, "...humans have diverse values depending on different cultures but also just by personal choice. So I believe in Value Pluralism. We just have to respect a lot of different values. And the question is what does that even mean to align to diverse values?" I agree Value Pluralism would be the right approach to AI alignment, but I'm wondering the answer to the question in her response if you want to take a stab?

Johan Falk
PRIORITY QUESTION!
Can a pair of particles be in a superposition of entangled and not-entangled? And are there any ways of doing a Bell's inequality-like test to test this?

Natalie Lines
Can you explain a bit about the emergence of spacetime from quantum mechanics and your paper 'Space from Hilbert Space' - I just serendipitously came across it and am fascinated by the concept!

James Tyler Canne
From what I understand, in quantum field theory every particle is really just an excitation within its field and there's a field for each type of particle. However, I've also heard that at high enough energy levels, the electromagnetic force, the weak nuclear force, and the strong nuclear force merge into one force. Does this mean that the fields that represent these forces actually merge into a single field or is it that the excitations in the fields become indistinguishable or is it something else entirely?

Matt Rogers
PRIORITY QUESTION: In his book review of Incomplete Nature, Daniel Dennett said that Terrence Deacon's theory of emergence (and specifically, how mind emerges from matter) is the best theory on the topic he has ever read. Since emergence is an area that you care about, I was wondering if you are familiar with Terrence Deacon's theory of emergence. If so then what do you think about it? Would you be willing to do a podcast interview with him as a guest, so we can hear you agree or disagree with him in public?

Paul Conti
Why is the topic of Climate Change so politically polarized in the U.S.A. ? We also have conservative Political Parties over here in Europe - the present governing party in the UK is actually called “The Conservative Party” - but all these parties accept the scientific basis of anthropogenic Climate Change. Yet, in the US, it seems to be split along Republican / Democrat Party lines, which seems most unhelpful.

Christian Hoffmann
i would like to know what's the set of things that quantum fluctuations can produce.
is it true that at every moment in time and for every possible matter distribution in space, there is a branch of the wave function, however small in weight, that contains that distribution?
i believe you did say that charge is conserved in all branches, so that would put at least one constraint on the matter content.

Omi
if we were to ever find a theory that would unify GR and QFT, do you think the equation for it would be something short (and elegant as Einstein would've called it) like the energy mass equivalence or do you think it would fill up pages with lengthy if-then-else type scenarios?

Kris V
Could you briefly talk about the Feynman Lectures on Physics? If I wanted to build toward a working knowledge base from forces to quantum field theory in my free time, would you recommend them as a good place to start?

Nalita S
I’m curious as to how you process reality. How does this thought make you feel… that all we can retrieve from reality is that we share entropy AKA life cycle with our loved ones… does it generate fear? Confusion? anger? Nothing? It feels like until we find the theory of Q gravity and going beyond QFT, something big is missing in our understanding of reality.

Leland Beaumont
I want to better understand emergence. Is the claim “life emerged from a chemical soup” any more rigorous, insightful, informative, specific, explanatory, or scientific than the claim “there were chemicals and then a miracle happened and now there is life”?

Eric Dovigi
We've heard of the “technological singularity.” But do you think there is such a thing as a “cultural singularity?” Or even just a hard cap on how complex human culture can become, as we accumulate more and more cultural artifacts and recorded history?

physics_kitten
I know that relativity treats gravity as curvature of spacetime, and I understand how that works in examples like the orbit of planets around the sun. But how do I make sense of a curvature of spacetime interpretation in the case of tides, which I think of as being caused by gravitational attraction of the oceans on the side of the earth nearer to the moon? What spacetime is curved, and is there a relativistic way to describe what is going on?

Tomáš Gajdoš
Can you talk about where the idea of Dark Energy comes from and what evidence supports it? You've talked about the evidence for existence and the properties of Dark Matter in your recent solo episode, which was great, but Dark Energy wasn't covered in that much detail and I'd love to learn about that too. Thanks!

Stefan Bernegger
How is entropy defined in an open system like the universe where total energy within an expanding volume is increasing and where gravity is ordering matter?

Shaun Virtue
How do we collide such small particles in accelerators? I can hardly throw a baseball in a straight line, I can't imagine the precision needed to collide two protons moving at 0.99c.

Aaron Bowden
Listening to your 'physics in crisis' solo edition, you mentioned the use of cut offs in effective field theory to avoid considering ultra violet energy levels in the equations. Can the same trick be applied to black holes and the big bang to avoid the singularity in relativity or am I asking a mathematically naïve question?

Samir
Could you please explain how it is possible to prepare a Qubit in a specific state (0 or 1) and then subsequently put it through a quantum gate resulting in the qubit now being in a superposition (eg a Hadamard Gate)

Brendan Hall
I’m having a very college student moment and have been moving towards socialist ideas and marxist critiques of society, and I realized that, though you had a great discussion on your moral and political opinions about animals and idw things, I don’t believe I’ve heard your thoughts on socialist subjects. Do you accept these sorts of criticisms of capitalism and perhaps accept any of the more radical opinions of socialist traditions?

Randall Bessinger
You have mentioned several times that you are not a big believer in heroes . Your view makes sense to me as I have been disappointed in some of my heroes over time. Did you always have this view or was it shaped by disappointing experiences.

Tim Converse
Especially in the context of talk about academic freedom and tenure, I'm curious what you can say from your own experience about pressures (whether subtle or overt) to be more narrow. and to what extent tenure exempts one from them.
For example, if you had had tenure at Caltech, would it have been easier to say "Hey, so my interests are turning in a more philosophical direction, so that's what I'm going to do now. Yay for tenure."
And are there contexts where breadth counts against you even other things equal? Like what about a physics tenure case with four great physics journal articles vs that same case with those same four articles plus two philosophy articles?

Mihai Todor
Recently, I learned that the Milky Way takes about 200 million years to perform one full revolution. It’s also estimated to be 13.6 billion years old. Dividing these numbers, I get 68 full revolutions so far and I find this number surprisingly small to obtain the seemingly complex and symmetric structure that it currently has. Do you have any insights into this? Is it possible that it revolved much faster around its axis earlier in time?

Robert Ruxandrescu
We know that black holes are maximum entropy in a region of spacetime. We also know that the 2nd law says that entropy overwhelmingly tends to go up, but has a very small chance of going down. How would this accidental, rare “going down” in entropy look like for a black hole? This would mean that the black hole could, just by accident and without emitting any Hawking radiation, end up being smaller, just by chance.

Andrew Jewell
I have a question about dark energy, and the long term expansion of the universe.
I've heard some people say that the galaxies will hold together, and just keep getting farther apart, until each galaxy is alone in it's observable universe.
But Neil deGrasse Tyson asserts that dark energy will affect ever decreasing scales, pulling apart galaxies and then planets and then atoms.
Are either of these characterizations correct?

Jim Murphy
I've been having an argument with a friend of mine. He believes that the general population is (on the whole) uninterested in philosophy and science, while I disagree. He claims that my perspective is biased by the people who I surround myself with. Namely, curious people who enjoy investigating challenging questions about reality. Do you believe that the "average person" is drawn to scientific inquiry and deep questions, or are most people blissfully ignorant?

Michael Lacy
Have you made any end-of-life plans? In particular, have you decided on a standard burial, cremation, green burial, or donating your body to a body farm or other research facility?

Andrej Chovanec
Is acceleration really an absolute quantity? I’ve thought that in the framework of the general relativity acceleration is relative, i.e there is no experiment to find out whether one is accelerating or not.

Jeffrey Segall
You mentioned writing about ideas in a fictional narrative. I was wondering if you ever saw the movie "My Dinner with Andre" and what you thought about it. I liked it (this was quite a while ago, I admit), while my wife hated it.

Nichael Cramer
I’d like to ask your advice about talking to non-physics-y friends.
A not-uncommon question is about wave-particle duality (typically phrased along the lines of “Have scientists made up their minds _yet_ if light is a particle or a wave?”)
A one-line answer I’ve found myself using is that “A reasonable way to think about it is that light —and other objects— travel about like a wave, but they interact with other objects like a particle”.
Does this seem like a reasonable (and not-too-badly misleading) way to go about this —at least at this level of discussion?

Sandro Stucki
In your reflections on the episode with Rosemary Braun you considered two possible explanations for why your guests often end up having a background in physics that you were not aware of when you invited them: (1) physics training is great and applicable in many domains, and (2) it's easier to come up with questions and make progress in other areas, so physicists move there. But what about (unconscious) familiarity bias? Could it not be that some of your prospective guests have that certain je ne sais quoi that subconsciously makes you more likely to invite them?

Ahmad Chaker
I heard in a accelerator like The LHC that we can’t detect neutrinos so we infer them from leftover energy from collisions, but how do we know it’s just a neutrino and not some other neutral particle in addition to that

Frederick Apollo
I hear you say the electron is point like but it also has an extent given by the Compton wavelength, how do we reconcile these two?

Dave Grundgeiger
In your September AMA, you said that humanity has the power to house everyone, but we're choosing not to. I agree 100%, but I don't think I know how to turn that into personal action that I can take. Do you have any ideas about how individual humans can leverage their individual power of choice to nudge collective humanity toward utopia?

Dave Grundgeiger
In your September AMA, you said that humanity has the power to house everyone, but we're choosing not to. I agree 100%, but I don't think I know how to turn that into personal action that I can take. Do you have any ideas about how individual humans can leverage their individual power of choice to nudge collective humanity toward utopia?

Roo Phillips
Is gravity really not a force and are physicists in denial about it?
In a tweet from David Deutsch on 9/28 he said the following about gravity :
"No force is exerted by gravity. If you hold out your arm, the only force on it is from your own muscles.
Very few people know this. I've known physicists to deny it."
Sabine Hossenfelder confirmed his tweet by replying:
"He is correct of course both in saying that gravity is not a force and that, a significant fraction of physicists deny it."

George
I was briefly introduced to eigenstates today in my chemistry lecture, and we were told that specific particles cannot exist at energies between eigenstates. Could you please elaborate on the exact reason why they cannot exist between these eigenstates?

Louis B.
Are there good theoretical reasons why we think a dark-matter-candidate WIMP should weakly interact, as opposed to being a non-interacting massive particle? Or is this a case of just being hopeful it interacts because otherwise finding it would be impossible.. so maybe we're looking for our lost keys under the streetlight?

Owe
I'm working through the older podcasts and just went through the conversation with Mike Brown (AKA Plutokiller). I got really curious and started reading up on the hypothetical Planet 9. One of the proposed theories is that it's not a planet but rather a primordial black hole. Putting aside the probability, my initial thought was one of concern as Black Holes can be pretty destructive objects. How dangerous would it be for there to be a planetary mass black hole within ~500 AU of the solar system?

Davis Yoshida
David Albert and Barry Loewer both expressed skepticism about the idea of self-locating probabilities. They both said something along the lines of needing to see more details about how these probabilities behave. Now that it's been a few years, do you feel their concerns have been answered? If so, where can I read more about the topic?

Katie Spaghetti
What are your thoughts on the philosophy and foundations of mathematics and the real numbers - i.e the continuum?
Do “real” numbers - infinite non-repeating decimals - exist in the real world?

Brandon Lewis
How do you feel about the new wave of fusion power startups? (e.g. Helion, First Light, MIT Spark, etc). Do any of them seem promising? As an outsider to the field, I can't tell which of the self-proclaimed "experts" to believe.

Ryan Vaughan
If the arrow of time arises from increasing entropy, why do we perceive time to advance at a constant rate (i.e. one second per second) when entropy increases at varying rates (say, inside a star vs. interstellar space)?

Russ Dill
In your paper with Bao and Singh on the Hilbert space of quantum gravity, you discuss the limited degrees of freedom of a local region of space and how it limits black holes. Does this also limit the compactness of the early universe? Additionally, would the special low entropy state of the early universe allow it to be more compact then if it were in some other state?

Pete Faulkner
In the context of the cosmological multiverse, I often hear people say things like "if we wait long enough" or "somewhere very far away" when discussing phenomena such as Boltzman Brains or exact doppelganger. If these phenomena are predicted due to purely random fluctuations then surely the likelihood of them happening in the distant future, or very far away are the same as them happening in the next five minutes, or close by (both incredibly small)?

The Memes of Destruction
How would you recommend a layperson conceptualize how time can become space and vice versa?

Sid Huff
PRIORITY QUESTION
Imagine two spaceships flying in parallel, some distance apart, say, 10 kilometers, or 100 kilometers. There is a super-strong long wire stretched between them. (Some sort of advanced technology may be necessary for this to work.) The ships fly on either side of a smallish black hole in space, such that the wire “cuts through” the black hole. What would happen to the wire and to the black hole?

Qubit
In your materialistic view of the world even morality emerges from physical laws on the most fundamental level. Do you think there is any chance of finding a moral principle, which attributes consciousness to some kind of exotic/ advanced computer, which does not have any human like properties? Certainly, we do not feel compassion towards such a device and it therefore seems hard to imagine why we would attribute consciousness to it anyway.

David Maxwell
Watching Oppenheimer, I found myself imagining what the world might be like had the US made theoretical physics an intergenerational priority. How much do you think the pace of advancement in theoretical physics is limited by the number of people doing it? Are there inherent limitations to the pace of change beyond the number of brain hours?

TheGreatDeceiver
A few years ago I was walking alone, at night hitchhiking in Argentina and was treated to the most wonderful night sky I’ve ever experienced, and probably ever will experience. I could see clear across the galaxy!
I’m wondering if you have ever been to SA to take in any of the amazing telescopes, like the VLT in Chile or others? what would you delight in peering at if you had full reign of one of those facilities!?

 

0:00:00.0 Sean Caroll: Hello everyone, and welcome to the October 2023 Ask Me Anything edition of The Mindscape podcast. I'm your host, Sean Caroll. This is yet again, what are those podcasts that is being recorded remotely. I'm on the road traveling. So, as usual, I'm gonna hope that the various audio quality issues don't get in the way, and I think this is gonna be pretty good though, I think I have a pretty good temporary mobile podcasting setup that should be fine, so I hope that's okay. The travel is an aspect, is one of the manifestations or one of the causes, I suppose I should say, that I have been very busy recently. Even more busy than my usual standards, which is why this AMA is coming out on the second Monday of the month. I just did not have the time to get it together. The AMA last week, usually the AMA comes out to the beginning of every month, unless there's some reason why some particular other podcasts should come out, then.

0:00:58.1 SC: Hope that's not too much of an inconvenience for anyone. There were a lot of questions, a lot of great questions, so I was kind of brutal cutting them down this time, apologies about that as always, no big news here, but one fun thing to mention is that part of the reason I was busy is not only traveling myself, but hosting other people coming in, and recently at Johns Hopkins, we had a visit from Scott Aronson, well-known theoretical computer scientist, quantum computing guy, complexity guy. And former Mindscape, guest. As well as, as some of you know, a collaborator of mine on one paper, we've done each other for a long time, but we only written one paper together, which was on the origin and evolution of complexity in the universe, the rough idea being that in a closed system entropy grows if you start with low entropy in a closed system, the entropy will increase with time until you equilibrate, but complexity first grows and then drinks, or it just doesn't grow at all, this is a good kind of open question.

0:02:00.6 SC: So the whole question of the origins of complexity and its relationship to statistical mechanics fascinates me, obviously, there's been a lot of work done on complex systems, features of complexity, things like that, but they generally presume from the start some kind of time directed-ness to the environment or the dynamics like energy is being inserted or something like that, there's some resource that is being used, so looking specifically closed systems, I think is a new thing and kind of interesting, so I forget how much of the story you all know, but years ago, Scott and I wrote this paper with Lauren Willett. No relationship to Jennifer Willett, to my wife, it was just an accident to coincidence, Lauren was at the time an undergraduate MIT, and what we did was we did some numerical simulations of cream mixing into coffee. Because cream and coffee mixing into each other as an example of how you can start with low entropy and simplicity, and evolve to high entropy and also simplicity passing through a complex phase.

0:03:05.2 SC: But we realize... Not we I have to say, we put the paper online and we were told by some e-mailers that in fact we made a mistake, we've had one person in particular, Brent Werness pointed out that we had made a mistake and he was completely correct about that, but it was actually good news because we fixed the mistake and the result with the mistake fixed is much more interesting than the result we thought we had originally, basically, some kinds of dynamics for the cream and coffee mixing together do not lead to complexity, other kinds do. And so now we have a new question that we can really try to get a hold on. How do you divide the set of dynamical laws covering the mixing that will and will not lead complex structures evolving. So we have some vague ideas, we have numerical simulations, and even though it's more than 10 years later, we're both busy people, he didn't have quite the home to pull it off. Now, Scott and I are finally finishing the paper and Red will be another co-author on it, and so we have all the new simulations done, we just have to write some text which we were doing while Scott was visiting.

0:04:16.3 SC: He also gave a talk. And it's great. So that will be coming out soon. And hopefully, I think leading to other research projects, we really, in some sense, look at the simplest, most naive definition of complexity, anyone who knows anything about complex systems, research knows that the idea of complexity is quite subtle and complicated, and you can go into details about what you mean by complexity? And there's different aspects, and we can think about how all of them might arise through the undirected distill elegiac evolution of Adams bumping into each other in a closed system, so I'm thinking about that as a future research direction and that's very exciting. Anyway, that's my little excuse for not getting the AMA earlier but it's all for a good cause. They come out eventually. Anyway, no one really loses anything. Remember that the Ask Me Anything questions every month are being asked by Patreon supporters of Mindscape, and you could easily be a Patreon supporter.

0:05:20.2 SC: All you have to do is go to patreon.com/seanmcarroll, and sign up to kick in a dollar or however much you want per podcast, and that's it. Then you can ask the questions and you also get ad free versions of the podcast, and also you get access to the low reflection videos and audios that I do after each podcast exclusively for Patreon supporters. So we appreciate that very much. And with that. Let's go. So start with Joe Callback, who says, "could you give a good update on the Mindscape scholarship recipients, which courses they've chosen, how they're progressing and so forth, it feels good knowing we're all supporting the thinkers of tomorrow with our donations." Right, this reminds me, there's sort of a good news, bad news situation here, the good news is we have a scholarship fund here at Mindscape, and you are welcome and encouraged to contribute to it, all you have to do is go to Bold.org/scholarships/mindscape. I think it's /scholarships. You can look it up. If you go to Bold.org and search for Mindscape, you will definitely find it. And the gimmick that we do is we try to give scholarships to people who wanna go to college and study the big pictures.

0:06:51.2 SC: Whether it is science or philosophy or math or whatever, something that is very academic and ambitious, and especially the scholarship is oriented towards people who are not already from... Their parents are professors and stuff, they've had. You wanna get new blood into the area, that's one of the hopes, and last year we were incredibly successful, we raised over $20000, and so we gave two scholarships worth $10000 each, which is not enough to cover an entire undergraduate education typically. But does make a difference. It does go a certain way, and it was Raymond Hassan and Liat Blaze that were the winners. I don't know how to pronounce Liat's last name, sorry about that, Blazes, Mallise? But we were very, very happy they both had extraordinarily good applications, and so that was a heart-warming thing that we did, the bad news is I have been very bad about keeping up with them, so Joy, your question is going to nudge me to catch up because we gave these out. I think that they were both still high school students, so hopefully they're both in college or university and doing whatever they wanna do.

0:07:58.8 SC: There's no requirements once you get the scholarship, by the way, if either one of them wants to go to university and then decide to study to be a veterinarian or a Wall Street banker. That is completely fine. We just wanna encourage people whose impressions at the moment or that they want to do the kind of thing that Mindscape specializes in, but then once you get it, you can do whatever you want, there's no reports, no annual updates or anything like that, but I'll try to chase down Raymond and Leon to ask how things are going. I hope that they're going very well, so that was a good question and... Yeah, and again, please feel free to donate. One thing that I can't figure out a way to do is how to thank the donors, there was one donor who recently gave $2500, which is amazing to me, which is wonderful. Again, warms my heart very, very much appreciated by the people who are ultimately gonna receive the scholarship. But I have no way to send them an email and say thank you, so here I go. Thank you very much.

0:09:01.0 SC: I don't know, I'm not gonna say whose name it was, I don't know if they want their name said, but I know who you are and I very, very much appreciate it. It's nice to think that we can do a little bit of good here at Mindscape world headquarters. Then Alex T asks, I've enjoyed listening to Mindscape for several years, and many of your AMA answers, you seem to hedge quite a bit, I've grown to appreciate your approach to truth and knowledge and how the precision of language complicates putting down objective fact. My question is, are there any subjects or statements that you would consider unequivocally true. Well, it's interesting, I don't know, I would have to be given an example of the hedging to understand better what it was that you had in mind about my hedging. I'm not saying that I don't do it, I'm sure you're right. Maybe what you're referring to is simply the fact that I'm always very reluctant to put zero or one as my credence in just about any proposition you might want to float out there. I think this is just a good, easy in practice, but if you're a practical question, Alex is, are there any subject or statements that you would consider unequivocally true?

0:10:05.6 SC: Well, we could be brains, in fact, we could be being taunted by an evil demon, all those skeptical scenarios that philosophers love to come up with are possible in my mind, so I don't like to consider things unequivocally true. On the other hand. There are some things that are sufficiently high credence that I'm not gonna worry about them. I'm not gonna bother to put a lot of intellectual effort or time into thinking about whether or not those things are true. They're true enough, they're sufficiently likely in my mind to be true that I don't worry about them, I move on with my life and try to build on them. And this can have, even that has sort of different levels of precision. General relativity is very accurately describing gravity in the solar system. I consider that a very, very, very high credence. For example, when I wake up tomorrow, gravity will still be attractive. Things like that, those are so high, I can't even imagine bothering, worried about, worrying about alternatives, but then there's the interesting gray area where I do think there is more than a 1% chance that the idea is wrong.

0:11:29.6 SC: But as a researcher who spends his life trying to think about ideas and develop them and push them forward, I nevertheless have to pick and choose what are the ideas I think are worth considering. So I'm actually more open-minded on the podcast than I am in my research career. I will talk to people who are not fans of the average interpretation, for example, I will talk to panpsychist about consciousness and so forth, but those are ideas that I put essentially no credence on as a practical matter, as a researcher, I think the dean of panpsychism has a chance, or Bowman mechanics has a chance or whatever, but I'm not being open-minded about them when I do my research in physics or philosophy, and this is... But been a weird situation that... Because I think that I do research in the foundations of quantum mechanics, but when you go to a conference on the foundations of quantum mechanics, there's an enormous amount of work being done by people who are studying not ever ready in approaches. And I think that that's perfectly okay, but I am just not an expert on that stuff, I'm not especially interested in that stuff, I just want to figure out how to use the paramedian approach to figure out how the universe works.

0:12:51.8 SC: And this is something I gained some clarity about. Over time, I remember back in the day when I was a graduate student, meeting someone who was a String theorist and he was working on cosmological implications of String theory, and I mentioned something I was working on that was not String theory-based, but cosmology was actually close to what he was doing, but it was not String theory-based, and he said. Yeah, I'm not especially interested in that if it's not String theory based, and at the time I thought that was outrageous, like, why are you so closed minded by it about this... But now I completely get it. I think that that's perfectly valid. It would be a big mistake if the community did that, it's very good that there are some people who are studying different alternatives, but as a single researcher you gotta pick your battles. You can't just think about every idea that's just not plausible and you're gonna be more effective if you focus your attention somewhere. So that's not exactly the question that Alex asked. But reminded me of that question, so that's the answer you got.

0:13:52.4 SC: Mark Bushey says, priority question. Does... Remember people who are AMA supporters get to ask these priority questions, which means once in their life, they can label a question by priority and then I will answer it, the answers might not be satisfying, in fact, I suspected there frequently not, but I will absolutely do my best to provide an answer to it. You only get to do it once, and so if you have a question that you absolutely want answered, try it out as a non-priority question first, and then if I don't answer it, just ask you this priority and I'll do my best. So Mark asks, does the block universe theory imply that motion is unreal or an illusion, and sometimes also implied that every moment is a kind of eternity, not a duration, but some other kind which I couldn't understand and that nothing happens in it. If yes. How does the thing we observe every day makes sense and to even have an illusion of some kind of motion seems to be necessary. What has it to do with eternity?

0:14:57.0 SC: Now the block universe theory does not imply that motion is unreal or an illusion. Motion means that you have something called position as a function of time. The block universe theory just says that every moment of time is real, that doesn't mean you don't have position as a function of time in the block universe at different times, things can be at different locations, that's what motion is the temptation to think of the block universe theory somehow implies that motion is an illusion or unreal comes from drawing a picture of the block universe theory and then pretending that it exists all at one moment of time, but it doesn't all exist at one moment of time. It just exists. I know that's a subtle distinction, so I won't say it again, the block universe theory or eternal-ism in the philosophy of time, does not say all moments of time are simultaneously real, it just says they're all real by definition. They are not simultaneously real and they are real at different moments of time, because they are different moments of time.

0:16:06.0 SC: Our language as usual is kind of shaky here, it's not really up to the task of describing these things well, but motion is fine that there's no problem with that. There's a closely related question In quantum gravity or even in some classical versions of physics where you drop time as a fundamental parameter and treated as emergent, even there, I would say that motion is real, it's illusory, but the steps to get there are tricky or I will admit that. We're not gonna do that right now. Peter and Caruso says that we could look at our universe from the outside... What would its shape be? Well, we don't know [laughter] So crucially, we can look at the universe from the outside for one thing or another thing, remember, I presume you're talking about the universe at one moment of time, right. To you talking about sort of a spatial slice through our universe, not we are just talking about the whole four-dimensional block universe picture, right? That's a whole another question, what would it be like to look at our whole space time from the outside, because we can't do that, we exist at individual moments of time, our world lines or the collection of all those moments, but at any one moment, we exist at just that moment.

0:17:22.5 SC: So I presume you're asking about the spatial slices through the universe, and in relativity, that's not even a uniquely defined question, we can't even say, Here is the right way to slice the universe, the spacetime of the universe in to moments of time. Once and for all, there's some freedom there, and that freedom might not make a big difference if you were here on earth. Spacetime is close to flat, everything's moving slowly compared to the speed of light, but when you're talking about the whole universe, then it begins to matter, okay. But those are some technicalities, there is also the well-known fact that if the universe is thought of as perfectly homogeneous and isotropic in space. Okay so homogeneous means the geometry is the same from point-to-point, and isotropic means the geometry is the same in every direction. Those are not exactly the same statement, only sound similar, because you could have, let's say, a magnetic field at every point in the universe, and that would have a direction at every point, so that would be the universe with a magnetic field that was uniform throughout, it would be homogeneous, same at every point.

0:18:32.7 SC: But they would not be isotropic 'cause would have a preferred direction, but if you have both, if you have homogeneity and isotropy, which the evidence is that we approximately do on large scales in the observable universe, then you only have three choices for the local kind of curvature that spacetime can have, namely spacetime can be flat, positively curved or negatively curved. That is to say a three-dimensional Euclidean geometry or a three-dimensional spherical geometry, or a three-dimensional hyperbolic geometry and instill all of those cases, the global completion of the geometry could be different. You can have a flat geometry locally, and that geometry could go on forever just have the infinitely big Euclidean three-dimensional space. Or you could be part of a tourist, you could be the Asteroids game, where when you go out one side, you come back the other one, so the universe could be three-dimensional tourist wrapping in on itself.

0:19:35.5 SC: The same thing holds true. If it's positively curved or negatively curved, you can have these weird topological ways the universe could close in on itself, the only rule is that if the curvature is positive everywhere, then the universe has to close in on itself, there are still different ways to do it, there's more than one global geometry, but they're all finite in size, if there's a minimum value of positive curvature everywhere, whereas if the universe has a space, I should say, has zero or negative curvature than it could either be final or infinite, we just don't know. But then there's another complication, I should say, one last thing there, which is that the data saying that empirically, the universe is close to flat, but there are air bars on that and close to flat is exactly when you don't know exactly what the answer is, right, because close to flat includes flat, almost flat, but actually positively curved and almost actually negative curve, both of those are allowed by the data, so we don't know, this is still the answer to your question. Finally, of course, when it comes to the universe, we only see part of it, we don't see the whole thing, so sometimes cosmologists talk about these models of the universe flat positively, negatively curved as if they are correct or as if they are the only option but that's flat out wrong.

0:21:00.5 SC: They only refer to the universe when it is homogeneous and isotropic. And it might very well be approximately homogeneous and isotropic within the observable bit, but while be different outside in particular, and ideas like eternal inflation, the universe is wildly different outside, so then the global geometry would just look like a chaotic mess, and there's really no more than I can say than that. So we truly don't know is the only good sensible answer I can give to that particular question. Carlos Nona says on the topic of misinformation, presumely referencing the Brandon and I in a podcast, we recently did. Do you agree with the idea that freedom of speech doesn't equal freedom of reach. What would you say to the freedom of speech fundamentalists that believe that any infringement of freedom is negative even when it prevents the spread of misinformation? Well. I kind of don't like simplistic mottos, like this freedom of speech doesn't equal freedom of reach. Of course, I'm sympathetic to the general idea, it's true, freedom of speech doesn't mean you can force people to listen to you, that would be absurd to think that.

0:22:11.3 SC: It doesn't mean that private corporations are necessarily obligated to carry your words if they don't want to do, there's nothing in either the legality of the first amendment here in the United States, or in moral or political or ethical philosophy that says that every company is obligated to amplify your speech that is just completely true and trivially true, but of course, it's complicated, and I think it's okay to be complicated, this is a... Freedom of speech as a topic is one where people really seem to not want to use their brains too much 'cause it might hurt them. This is a topic where you have to think about things carefully, there are balancing values that we have, we want everybody to be able to speak. We want everybody to be able to say their minds, we do not want to pass laws or regulations that prevent people from talking, but that doesn't mean that they can talk in any venue they want with any loudness they want, right.

0:23:18.0 SC: So we have other values that we have in our society, misinformation, just misinformation can be as simple as shouting fire in a crowded theater, for example, or it could be more blatant lies, every country has its libel laws and its defamation laws and things like that. These are restrictions on speech, you can say things, but then you'll be punished for them and that's okay, that's perfectly 100% okay, and it's hard to draw the line exactly. So the very first step in thinking sensibly about freedom of speech is to admit that we have to sit down and think about it hard. That we can't just boil it down to mottos, so that doesn't give us a simple answer. Yeah, that Philosophy doesn't let us immediately go from that to saying, okay, therefore the following restrictions on misinformation are allowed. I think it's a very tricky question, I'm much more in favor of freedom of speech than the average person, I'm completely on board. If some university group wants to invite some terrible, terrible, terrible person to give a talk, and I say that online, and some people say, yes, I completely agree with you, except for the really terrible people.

0:24:37.0 SC: And my response is no, I actually... Even the really terrible people, if some individuals wanna listen to a really terrible person. I think that should be allowed, others should not be forced to listen to them. I do not believe that you should hackle people off the stage or anything like that, you can just not go, you can just counter-program, you can set up your own speech that says things that explain why the person is wrong. I think that's the right way to do it again. It's complicated. You don't want people coming and inciting violence, for example, and there are going to be gray areas, and I think again, yet again, I'm gonna keep saying it again and again, 'cause people don't believe me. You have to think hard about the gray areas, and maybe you have to even admit that they will remain a little gray, they will never be cleared up by some simple model that makes everything cut and dried.

0:25:31.9 SC: Blake Brazier says. I'm an electrical engineer by trade. And I've often thought it is a terrible historical mistake that the charge of the electron was decided to be negative instead of positive. My question is, do you think electromagnetism and electrical engineering would be easier for people to comprehend if we all agree to swap polarities on all things carrying electrical charge? Such that anything positive would be negative going forward and vice versa, or is there some fundamental reason the charge of the electron should be negative and not positive that I'm simply not understanding. I don't think either one of those is true. Blake, sorry about that. There's no fundamental reason the charge on the electron should be negative, it goes back to the days when they were first studying electricity, but I had no idea there were things called electrons. They discovered electricity long before they knew that what electricity was the motion of light elementary particles that we now call electrons. So they had to choose, what was positive? What was negative? That they made a choice. 50-50 chance. They got it wrong.

0:26:33.8 SC: It absolutely would be more convenient if the electrical charge are defined by convention to be positive rather than negative, however, it is not so much more convenient that it's worth switching, there are costs to changing a convention that everyone agrees on, just ask any country that is switched from imperial weights and measures to metric, right? In the case of going from imperial to metric, there are clear benefits that are pretty big, dividing by 10 is still a lot easier than doing some of the wacky things that imperial units demand that we do, so I think that the case could be easily made that metric is just better, but the positive sign versus the negative sign in electromagnetism and electricity in particular, the charge of the electron is a relatively minor thing. You can handle it. So if to change it would cause a whole bunch of headaches because we had books, we had papers that are written with the existing sign convention. And if you started the idea that going forward, all books and papers have a different sign convention, they're suddenly incompatible with each other. And a million other little tiny annoying worries, I guarantee you, the headaches that would come along with changing the convention are noticeably bigger than the benefit you would get from the ultimate change having been accomplished.

0:28:00.4 SC: Michael Frid says, I was under the impression that when a lot of matter occupies a very small space, a black hole will form, when I hear about the Big Bang, it sounds like all the matter in the universe was compressed into a very small space. I was under the impression that under our current understanding of the laws of physics that much matter in such a small space must create a black hole, how was all the matter and energy in the universe able to escape such a small space preventing a black hole from forming. Good I like this question 'cause it's easy. You are not correct, you are not under the right impression, it's not your fault, 'cause I'm sure you've been told this, but it is not correct to say that when a lot of matter occupies a very small space, a black hole will form as a necessary conclusion and there's even nice little bit of thought that you can put into it that explains why that's true. Namely, general relativity as a theory of physics is invariant under time reversal, that is to say any solution that we have to general relativity has implies the existence of another solution which is just the same solution, but running backward in time, okay.

0:29:10.1 SC: You know this from regular old Manton in mechanics, if there's a pendulum rocking back and forth, you can rock back and forth backwards, in fact, it looks exactly the same if you throw a baseball one way, you could throw it backwards along the same trajectory. It is also a good solution, so if you imagine a perfectly legitimate solution to the equations of general relativity of the form, I have a planet or a star and it collapses to make a black hole. Okay then you know that there's another solution that says there is the time reverse of a black hole and it spits out a planet or a star.

0:29:47.7 SC: Now, there is something called the arrow of time. Entropy increases with time. So even though, strictly speaking, at the microscopic level, such solutions to the equations must exist, that doesn't mean they actually happen in the real world, there are solutions where scrambled eggs un-scramble or cream and coffee unmixed from each other, that doesn't mean that they're realistically going to happen, but there are solutions to the equations, and therefore the statement that if there's a certain amount of matter and space, there must be a black hole in its future. Can't be right. Because maybe there is a white hole, which is the opposite of a black hole the time reverse of a black hole in its past, Carlo Valley, one of the very first Mindscape gas just came out with a new book or maybe is coming out soon, called white holes. If you want to read more about white holes, he thinks they might actually exist and I'm a little more skeptical about that. So the Big Bang.

0:30:45.1 SC: As a solution to the equations of general relativity is very, very, very much like a white hole, it has a singularity in the past rather than the future. And matter comes out of it rather than falling into it. So the whole situation is 100% okay. If you want a less technical, more hand, we give yourself a warm and fuzzy feeling explanation, the universe was expanding very rapidly after the Big Bang, the matter was all moving apart from everything else, it was moving apart so fast that did not form a black hole. If you like that explanation, better go nuts with it, but one way or the other, Nothing whatsoever is being violated as far as laws of physics are concerned. Ryan Santos says I feel like the popular speculations about extraterrestrial life are too constrained, do you think there is any special reason to presume that life elsewhere is necessarily planet-bound and subject to restraints like hours? No, I don't think there's any necessary reason to presume that if you put the word necessarily in there, then the answer is gonna be no. Life can be lots of different things. I think it was... Oh my goodness, Fred Hoyle the astro physicist who was a champion of the Steady-state theory of the universe, who wrote a book about life in a giant molecular cloud, had nothing to do with living on a planet whatsoever. Robert Forward, of course, wrote story about life on a neutron star.

0:32:13.6 SC: So I think you can be very open-minded about the possibilities, but I would say two things, number one, we know life can exist on a planet, we know a lot about Halla can exist on planets. So given that the space of possibilities is very, very, very big, I think it makes perfect scientific sense to mostly focus on the regimes in which we know a little bit about what might happen, and it's perfectly legitimate scientific practice, I think. And secondly, I do think that there are reasons to think that planets serve as good environments for life forming, we don't know a lot about the original life, etcetera, but it's easy for us to imagine the planets have the right kind of environment, the right kinds of conditions for life to form, whereas in other more extreme environments, you really have to stretch yourself a little bit, so even though it's not necessarily true that life should be planet-bound, it wouldn't be surprising to me if most or all the life that we find elsewhere, turns out to be. Tim Jianitos. Jianitos. Sorry Tim, I'm just not doing a good job with your name today, there's a famous paper by John Wheeler where he makes some bold statements and argues that they are these bold statements are basically irrefutable.

0:33:36.3 SC: What is the stance of the modern physics community on them, the world can't just be a machine ruled by continuum, physical law, there is no such thing at the microscopic level as space time and bits are the most fundamental reality. Well, I think that all of these are quite reputable [chuckle] at least if what you mean by that is we can coherently sensibly imagine that they are not true or true in any useful way. John Wheeler was a brilliant physicist, and he made a lot of good contributions, he was also quite fond of exaggerating and he was also quite fond of poetic language. So sometimes his statements are not clear enough for us to even really say whether they're right or wrong, so when you say things like, the world can't just be a machine ruled by continuum physical law. I never knew the world was a machine ruled by continuum physical law, so for one thing, what do you mean by machine in particular, do you mean it's mechanistic. To me, is deterministic. Do you mean it is non-teleological, do you mean there are no base rules. I have no idea what that word is supposed to mean in that context. Maybe the relevant world word in sentence is continuum. Maybe he is being skeptical about the real numbers as opposed to the integers about smooth quantities rather than discrete quantities.

0:35:01.3 SC: I think in that case, it's absolutely unknown whether or not the world could be completely continuum or whether or not it could be and is discrete, I think we should be open-minded about that. Whether there is such a thing of the microscopic level as spacetime, I don't think that there is, but there could be... Come on, how do we know these things? And bits are the most fundamental reality, how do you know what the most fundamental reality is. How does John Wheeler know that? I think that this one is the closest I would say to just being wrong, bits are the most fundamental reality... I think that bits are descriptions of reality, numbers are not the most fundamental reality numbers are ways that we use ideas that we appeal to when we describe reality. There is a paper I wrote that is online, you can find it simply called Reality realism, I think I mentioned it here before, the idea being that physical reality is real, nothing else is quite as real as physical reality, things like energy and information and mathematics are all different properties and descriptions of the underlying physical reality that might turn out to be a bad point of view, the point of view that I just advocated right there, I'm open minded about that, but let's not be the ahead of ourselves, let's think of these as provocative statements that make us think, not things that are basically irrefutable.

0:36:33.0 SC: George Hampton says, "What led you to write these new books? Did the pandemic have much to do with it?" Yeah, the pandemic had a lot to do with it. I did videos on the biggest ideas in the Universe series. You can find them on YouTube. Something like 48 videos of me sitting at my iPad and writing out little pictures and little equations and trying to explain physics. And since I had done that much work, along the way, I thought it would be relatively useful and straightforward to turn them into books.

0:37:04.4 SC: Famous last words, of course, it's been an enormous amount of work. I have two of the books written. One more to go. And the reason why it's hard is because when you're writing a book, there's two things. The whole gimmick here was that I was trying to be a little bit more detailed than I usually get. A little more quantitative. There are their equations, stuff like that. And that's fine, but it is a different kind of mode to be writing in. The purely technical mode where you're writing for students who are in a curriculum and have had certain prerequisites and are guaranteed to be invested and incentivized to do the problem sets and stuff like that, that I can do. That I've done. That's a very standard way of writing. Physicists... Any person in a technical field knows how to do that. And the popular way of writing where you don't have equations, where you're trying to make things as understandable as you can without reference to all the detailed quantitative equations, but other statements about the theory definitions and things like that, that I know how to do too. [chuckle]

0:38:13.9 SC: So it was too easy in writing the new books to lapse into a purely technical way of talking. Well, you've seen this equation before. Now let's put it to work. And that's just hard. I don't shy away from showing the equations, but it has never been true, even for the most bright physics graduate student, that you just see an equation and you instantly internalize it. Even if you understand what all the symbols mean, the implications of it, the way you should think about it without explicitly solving it, all of that stuff is an intuition that you build up over time and practice. And in the course of a very short book, in the new book, Quanta and Fields, we go very deep into Quantum Field theory. Quantum Mechanics, all by itself, is, I forget, either just three or four chapters at the beginning. It's pretty quick. And then we go right into field theory, Feynman diagrams, renormalization, symmetry breaking, confinement, effective field theories, the spin-statistics theorem. All this stuff is covered in a remarkably short period of time. So the real, real, real challenge was including enough the details that it was useful and didn't seem like cheating without relying on the, obviously, incorrect assumption that I could just show people an equation, explain it and they would instantly get it and we could move on. You have to be able to relax, catch your breath, think about it, really think things through a little bit.

0:39:47.6 SC: Anyway, that's not the question you asked, but it is an extra challenge that arose in writing these books. And I think that was part of the motivation, because I wanted to do something different. These books are purely pedagogical. I am not pushing any new agenda or any new theory or breaking any new ground on our understanding of the universe. And usually when I write books, I am, or at least I am trying to make an argument for something that you are welcome to disagree with, whether that argument is about the arrow of time or the foundations of quantum mechanics, or the underlying ontology of philosophical naturalism. The books that I like to write are ones that take a stance and say something new. So this book, these books, this series of purely pedagogical books is something different. And I wouldn't have done it if I couldn't do something that I thought was different, do it in a different way. So even though the content is not new and it's mostly decades or centuries old, the way that I'm trying to do it is new, and that made it interesting for me.

0:40:54.8 SC: Fran Pla says, "In Episode Number 41 with Steven Strogatz, you mentioned that you were taking melatonin when you were in different time zones. So to connect it to Episode 251 with jet lag, eating early and daylight is good measures, adjust our internal clocks. My question is, are you still taking melatonin to combat jet lag?"

0:41:15.4 SC: Well, I haven't taken it in a while. I haven't been doing that much international travel in a while. So I didn't really need to do it. But I would take it. Yeah, I have no objections to doing it. I think that my experience of melatonin has been very good. I generally will take it the first couple of days if I'm doing a long trip that I'm going very far away, and either to... Depending on which direction I've gone in, to fall asleep at bedtime or just to, otherwise, shift one's biological clock. Of course, there are other things you can also do; eating early. Daylight is something that everyone knows. I hope that seeing daylight, especially in morning times, is a useful way to combat jet lag, to get your body somehow adjusted to the fact that the sun is up, even though it thinks that it's supposed to be night time. But look, whatever works for you. Everyone's biology is different. Everyone's psychology is different. I suspect that psychology also matters. As we learned with Rosemary Braun, lots of things matter. There are deeply interconnected networks of both cells talking to each other and internal structures inside cells which matter for all this stuff. So not surprising to me that different people react to different strategies.

0:42:33.6 SC: Stevie CPW says, "If I read the report correctly, 30 billion suns could fit into a recently discovered black hole. How does a black hole get to be that big? And when you learn of such discoveries, do you have any emotional reaction to their awesomeness, or is it more a matter of fact to you?" So I bet that what the report said was that the mass of the black hole was 30 billion times the mass of the sun. That is allowed, but at the upper level of what we know for what we call supermassive black holes, which typically appear at the center of large galaxies. And the physical size of a 30 billion solar mass black hole would actually be much smaller than 30 billion times the size of the sun, because compared to a black hole, the sun is a big, not very dense thing. 30 billion of them would be much larger than a black hole. So when you squeeze it together, then you get a black hole. But if the question is, "How does a black hole get to be 30 billion times the mass of the sun?" Yeah, I don't know. [chuckle] We don't know.

0:43:41.5 SC: Keep in mind, a large galaxy will have trillions of times the mass of the sun in total in the mass of the stars, plus the dark matter. Many trillions of times the mass of the sun. So 30 billion times the mass of the sun is big. But it's not wildly crazy big compared to the size of the galaxy, it's actually small. So there are very good questions to be asked about the formation mechanism of these black holes. Maybe there's something weird and fun about primordial physics or inflation that helps see these black holes, and galaxies later form around them, or maybe there is just some efficient mechanism for converting gas and dust into the black hole to sort of spiral it in there, focus it, or funnel it in to the black hole to make them heavy. I don't know. This is just not my area whatsoever, so hoping these astronomers can figure that one out.

0:44:36.1 SC: Gatey Lee Smullen, which, against my own advice to myself, I'm going to guess is not someone's real name. But that's okay. "As a good Bayesian, how do you differentiate belief, hope, and faith?" Well, these are very different words. I think it's pretty easy to differentiate them no matter what. Faith is a word I just don't use as a good Bayesian. People do use the word. If you try to ask them what they mean by it, different people will have different things to say, different definitions of it. Some of them will be a little slippery. So I just prefer myself not to use it, because it's one of those words that different people will insist that you must have in mind the same definition that they do, which I typically don't. So I just... I'm not gonna use it.

0:45:21.7 SC: Belief, I use all the time. I think belief is fine. I think that people sometimes think the word belief should be defined as belief without evidence, but I don't think that's right. [chuckle] Again, when I say right, right and wrong are not words that one attaches to definitions in any sense of there being an objectively right definition to use, but I do think that some definitions make more sense and are more useful and are carrying more information, and so forth. So I like to use the word belief when I believe things. And I can believe things with different levels of credence. I can believe things because I have good evidence to believe them. I believe that I had a chicken sandwich for lunch today because I have a memory of it, and I have no reason to think that that memory is faulty. Beliefs would typically have credences attached to them, so they're not yes or no. You can say, "Well, I believe that it's probably true that something is true." I think that's fine. Hope, that's a completely different word. Hope is an expectation for the future, not something you do attach to propositions at the current moment. So I can hope that the Philadelphia 76ers win the NBA championship. That is not to say that I believe it will happen. Certainly not to say that I have faith in it happening.

0:46:43.6 SC: JC says, "So Google is shutting down the podcast application and asking us to listen to podcasts in YouTube. I don't like YouTube for podcasts. I understand that you podcasters receive less credits for YouTube videos than for actual podcasts. Where do you suggest we go now to listen to you?" I don't even know Google had a... I mean, probably, I knew that Google had a podcast application. Google, as useful as it is, it's becoming less useful. And certainly one of the reasons that as a company, it's losing the credibility among customers is they keep offering new products and then cancelling them after people get used to them, which is very annoying. So I guess that's gonna happen to a podcast reader, but there are lots of podcast readers you can use. I personally use Overcast for my mobile devices, for my phone or laptop or whatever. If you go to the Mindscape Podcast web page, which is preposterousuniverse.com/podcast, and you look on the sidebar over on the right...

0:47:51.8 SC: By the way, it's actually very nice to go to The Mindscape Podcast web page. I'm looking at it right now as I speak, and I'm realizing I gotta update this web page a little bit. I try to put advertisers on there, and I haven't updated them in a while. Sorry, advertisers. [chuckle] They send me their money, but I do not always serve them correctly in response. So I'm gonna have to update that. But one of the things you will find on there is a bunch of ways that you can listen or subscribe. ITunes, Stitcher. There it is, Google Podcasts. But also Spotify, TuneIn, Podbay, Podtail, Podchaser. Whatever. There's lots of different ways to do it. So you have both a service that will feed you the RSS feed, and then you also have a reader, like Overcast or something like that. I think you would do much better to just Google what the different options are, or search for what the different options are. I don't know. Ask ChatGPT what the different options are, rather than asking me. I don't claim to be an expert in comparing different podcast readers.

0:48:52.7 SC: Arie Maudie says, "Is it possible that at the very center of a black hole is the same thing as the universe before the Big Bang?" Well, this is one of those questions I have to think about what is being meant here. For one thing, if you know a little bit about black holes, the center is probably not what you're thinking of. If you carefully study what happens inside a black hole, there's a Singularity. But the Singularity is not at the center, the Singularity is in the future of you when you fall into the black hole. The correct statement is, once you're in the black hole, moving toward the Singularity is moving forward in time. So you hit it in the same way that you hit tomorrow, not in the same way you hit the wall to the left or right of you. The center of the black hole is whatever made the black hole. If there was a star that collapsed to make the black hole, then if you can get to the center before hitting the Singularity, you will see the tiny squeezed-together star there, if you could possibly get there. So probably what you mean is, Is there something that at the Singularity resembles the universe before the Big Bang?

0:50:03.0 SC: So two things to say about that. One, we have no idea what was the universe before the Big Bang, or whether there was any universe before the Big Bang. Maybe there was. That's something to put a lot of thought into, write papers about, think carefully about. I do that. Other people do that. It's an ongoing thing. But the Big Bang may have been the beginning. There might not be any such thing as the universe before the Big Bang. Or there may have been. We just don't know.

0:50:27.7 SC: The other is, there's zero reason in known physics to think that there's anything beyond the Singularity of the black hole. There are reasons to think there is no Singularity to the black hole, namely that there is quantum gravity. And the prediction of a Singularity in the future of the black hole, once you're inside the event horizon, comes from Classical General Relativity. And we know Classical General Relativity is not going to be right. There's going to be something quantum there. Probably something finite in one way or another. We just don't know what exactly that way is. Now, people have absolutely speculated about the possibility that when you make a black hole at the center, somehow it creates a new universe, or it pops into a new universe, or transitions into a new universe. It's a free country. You are welcome to speculate about such things. But we don't have any theory that says that should happen, or does happen, or any knowledge of exactly how it would happen other than waving our hands a little bit about quantum mechanics, gravity. Who knows, really.

0:51:35.5 SC: Anthony Nault says, "What is the difference between the wavefunction realism of the kind that David Albert talks about versus Mad-Dog Everettianism?" Yeah, so this is one for the experts. And by the way, I know that this is a mixed audience listening to the AMAs. And sometimes, for very predictable reasons, people express a frustration that some of the answers to some of the questions are not precisely on their particular level of knowledge. That is to say some answers are too technical and presume that you know something about what's going on, and some answers are too simple, or you've heard them before and you don't wanna hear this basic stuff over and over again.

0:52:22.6 SC: What I try to do is calibrate the answer to the question based on the question. Usually from the question and how it's asked, you can figure out something about how much the person knows about the stuff, how technically informed they are about it, and you'll try to answer on that level. But also, I might be guessing wrong and I might be assuming that people listen to previous podcasts and stuff like that, so my apologies if I don't get it exactly right. But also, my advice is live with it. [chuckle] As the listener, you're gonna listen to some things that are not exactly on your level. Some will be below them in terms of your total knowledge. Some will be above them. All that is fine. Think about how you would answer it if it's too simple for you, and sit back and listen if it's too high level for you, and think about the day when it will be too low level for you because you've learned so much that you've completely caught up.

0:53:18.8 SC: So this question is a relatively high level question. This is an ongoing sort of frontier thing in foundations of physics. Wavefunction realism is something that David Albert, former Mindscape cast, talks about. Other people also talk about it. And it sounds very similar, absolutely, to Mad-Dog Everettianism, or just Hilbert-space fundamentalism, you will sometimes hear it called. This is something that I have promulgated and supported, the idea that the right way to think about the fundamental nature of physical reality is that it is something, it is physical reality, and it is modeled by a vector in Hilbert-space, and that is the right quantum mechanical lesson to draw. The wavefunction realism sounds very similar to that because we often casually trade off the word vector in a Hilbert-space, or a quantum state, or a state vector, any of those are equivalent, for phrases like the wavefunction of the universe.

0:54:18.9 SC: And that's okay if you know what you're doing, but in this case, there is a subtle difference, and the subtle difference is the wavefunction that David is talking about is a specific representation of the physical state vector that I would talk about, in particular, is a representation in configuration space. So you know that if you have one particle that has a position X, its wavefunction is psi of X. Some complex number for every value of the position variable. If you have multiple particles, with possible positions, X1, X2, X3, etcetera, you have one big wavefunction that is a function of X1, X2, and x 3, and whatever other particles you have, you can always do things like change variables. You could, for example, go to momentum space and talk about psi of P1, P2, P3, or any other different kinds of things. You could work in the energy eigenbasis, etcetera.

0:55:15.9 SC: To the Mad-Dog Everettian, who cares? What's real is the vector. What's real is not the basis that you use to express that vector in. There's nothing more real about the position space basis or configuration space basis than there is the energy eigenbasis, or the momentum eigenbasis, or what have you. But to wavefunction realists, there is. That's the difference. They want to say that the set of all the positions of all the particles, which is what you're building your wavefunction out of, has some reality to it. And that's the difference. And I see no reason to think that it has any reality to it. It doesn't even really fit very comfortably with the true fact that modern physics works with fields rather than with particles, right? You can gussy it up to do that. You can say, "Well, it's psi the wavefunction as a function of all of the field profiles in the space, etcetera." But to me, or from my point of view, from the Mad-Dog Everettian point of view, it is giving some ontological ump to good old ordinary three-dimensional space in ways that are not really supported by anything that we know in physics. It's convenient, but I don't see why we should give it any special preference. There's no difference to me versus a wavefunction in position space or momentum space. Why is one supposed to be better than the other? I don't know. What matters to me is the overall vector.

0:56:48.3 SC: Of course, I should say, that means more work for the Mad-Dog Everettian, because we have to explain why space is so useful. But that's okay. We do that. That's part of our project for writing papers. "Only normal person asks a priority question. In an earlier AMA, you stated that in contrast to suicide, euthanasia may be allowable, which would indicate that you think there should be general moral and/or legal prohibition on suicide. At 30, I've come to the conclusion that, number one, I'm generally unhappy due to depression, rather than external variables. Number two, I'm completely skeptical that I make people around me happier than if I were to not exist. There are currently living people who would be saddened by my death, but on some level, I feel ultimately hope that I can find an opportunity to check out early, maybe within 10 years, 15 years at the latest, without anyone feeling harmed, which will certainly require certain behavioral commitments, but at least seems like an objectively non-futile goal, possibly the only one that is possible in a world where we have pretty limited agency to make things better for others. And the Second Law of Thermodynamics implies existence will always be more destructive than constructive. Do you think creating and working toward harm-mitigated suicide as a long-term plan is ethically and logically sound, or would you pause it some objection?"

0:58:11.6 SC: Well, this is a tough question. This is a deep one. I'll try to take it as seriously as I can here. I honestly don't recall in an earlier AMA saying that in contrast to suicide, euthanasia may be allowable. But maybe I did. I don't exactly remember the specific words I used there. I think that there are two different things going on. There are more than two. But at least, two big, big, big considerations here. One is, is there a general philosophical ethical prohibition against committing suicide? And then in that, I would say no. I don't think that there is. I think that I would want individual people to make the ultimate choices about the fate of their lives to the extent that is possible. But the other thing is that there are practical considerations here, and I can't downplay them. It's somewhat loosely analogous to the free speech discussion that we had before. The practicalities matter. And there are many practicalities, and I'm sure that you have been exposed to them. I'm probably not telling you anything you don't know, but people change their minds about committing suicide. Things like depression can be dealt with with some combination of therapy and medication and things like that. And suicide is one way. Killing yourself is not something you can undo.

0:59:38.1 SC: There's a famous story that is told somewhat jokingly, although it's not really something to joke about. Murray Gell-Mann, the famous physicist, one of the best physicists of the 20th century, when he was in high school, he very seriously contemplated committing suicide, and it was about when he had been accepted to go to college at MIT. And he told a joke that he realized that going to MIT and committing suicide do not commute, which is a physicist way of saying, doing one than the other is not the same as doing the other than the one. You can go to MIT and then commit suicide, but you can't commit suicide and then go to MIT. Which is a way of saying that things can change. They changed for Murray Gell-Mann. That doesn't mean they will change for you. I can't make any promises like that. And again, I'm sure that you've heard things like this before, but I have to get them on the table because they're true, and they're important to keep in mind.

1:00:33.9 SC: I would disagree a little bit with where you say the Second Law of Thermodynamics implies existence will always be more destructive than constructive. That's not what the Second Law of Thermodynamics says. The Second Law says that entropy increases. I started this podcast with talking about the paper I'm writing with Scott Aaronson about complexity and how it evolves over time. And the point is that complexity increases even though entropy is increasing, at least for a while. There's some cosmic horizon, billions and billions and billions of years in the future, where it's gonna be hard for complex structures to persist. But right now, as far as the laws of physics are concerned, yes, entropy is going up, but that does absolutely not equate to being more destructive. There's all sorts of things that can be built, that can be created, can be brought into existence that have never existed before. There is absolutely a huge amount of room for human creativity and constructive gifts to the world that can come from people doing good things despite the Second Law of Thermodynamics being at work.

1:01:49.8 SC: So I don't agree with the general idea that life is not worth living or anything like that. To me, life is very worth living. I very much recognize that other people... Everyone has their own circumstances. I respect that. I'm not gonna tell you that your life, in particular, has to be worth living and you're making a mistake and I know better than you do, because I don't. But I don't think that it's right to look at these sort of cosmic tendencies or laws of physics as justifications or arguments in favor of this or that, whichever position you might have, exactly because of the finality of it, because of the ultimate nature of it. I think that it is absolutely sensible to be as super duper cautious and thoughtful as you can possibly be when it comes to questions like your own life, for whatever to do. Not just suicide, but even risky behaviors and then things like that. You only get one life. I don't think that there's any afterlife or anything like that, so there's an enormous presumption against ending it, unless you really, really, really, really, really, really don't see any other way. Especially because as you say, there are people who would be saddened by your death, you hope to make that not true down the road in some number of years, that sounds hard for me to see how that could happen. I think that there will always be people who are saddened when other people... Their lives end one way or another.

1:03:26.7 SC: So again, philosophically, I think that it should be possible to do something like that, but the practicalities are so large. And I don't quite align with some of your philosophical statements in the question, so I would... If nothing else suggests working as hard as you legitimately can to give a sincere thought to other possible strategies out there, I'm not the person to be listening to about these things, I'm not a professional in any way. You should not take this as medical advice. I'm just giving you my AMA level opinion, which is not even very carefully researched or anything like that. So these are tough questions. I sympathize with your situation. We don't, as a society, make these questions very easy. We don't make them... We don't talk about them in an adult way. One of the first podcasts I did with Megan Rosenbloom was on death with dignity. And usually, that's in the context of euthanasia, but it's euthanasia that obviously the person has signed off on, usually in cases of severe disease or something like that.

1:04:46.5 SC: And there, I do think... I would say everything I just said about the seriousness of the issue and the importance of really thinking things through, but if you have a terminal disease, then I'm more on the side of do what is gonna be most comfortable for you and pain-free. Whereas if you think that you're physically pretty put together, etcetera, then the presumption, I think, goes the other way. But as I said, everyone's situation is different, so all I can do is wish you my best of luck.

1:05:18.9 SC: Now, I'm going to group together two questions. Robert Grenazay asks a priority question. "Is there any credence to the idea that dark matter is space that exists at a level of higher dimensionality, so we cannot see or interact with it?" And Lars Willert says, "We say there are four fundamental forces, although Einstein showed us that gravitation is merely a distortion of space time. Could the other three forces also be considered to be just distortions of dimensions? And might these dimensions be correlated to the extra dimensions referred to, in String Theory, as curled-up dimensions?" So both of these have to do with extra dimensions and their dynamics in some way. So for Robert's question, I want to try to figure out what exactly is meant by "Dark matter is space that exists at a level of higher dimensionality." I think that the respectable version of that that has been explored by physicists is if we have extra dimensions of space and they are curled up, then you can have particles with momentum in the direction of the extra dimensions, particles that are doing little loops in the extra-dimensional dimensions. And how those particles appear to us is just as regular particles, but with more mass than they would otherwise have. So they're heavier. So these are called Kaluza-Klein particles, after Kaluza and Klein, who invented the idea, or pioneered the idea, anyway, of extra dimensions of space.

1:06:54.6 SC: But really, they're particles at the end of the day, so I'm thinking that this is maybe not what you have in mind. So yes, you can imagine particles that somehow move in extra dimensions and that affects their properties, but not in a very fundamental way, not in a very profound and different way. Ultimately, they're just particles with some masses. But the point is that that's what you want to be the dark matter. The dark matter that we know of, we know a lot about its properties, actually. We know that it doesn't collide with itself. We at least not noticeably so. We know that it's cold, so it doesn't move near the speed of light. We know its density, we know its initial distribution in the early universe, and we know a lot about how it has evolved since then, and so on and so on. And in all of these ways, it acts like a particle, like a good old particle weekly interacting with some mass.

1:07:52.0 SC: So I wouldn't advocate working too hard to invent models that are nothing at all particle-like. You want to invent models that just reproduce ordinary particle behaviors. They might be interestingly different in some ways, but when you think about... Both questions are asking about Could dark matter be some feature of space itself? I don't exactly know what that would mean, but it would strike me as weird if a feature of space itself had all of the particle-like behavior that we normally attribute to dark matter. So it's not a natural or easy kind of thing to think about, although maybe. At the end of the day, you can always just say maybe.

1:08:35.4 SC: Brian Mapes says, "Do you have more specific ideas or examples about how to use fully Tully macro-scale entropy, parenthesis, complexity, separately from the abyss of molecular entropy, which increases by the Second Law? Inspired by the coffee figure in The Big Picture. By the way, in The Big Picture, I plot the coffee versus cream example that Scott Aaronson and I are trying to finish our paper about. That's what Brian is referring to here. "Inspired by that in your Berkeley colloquium last year. I'm trying to do this for meso-scale patterns or configurations of clouds and weather scenes, which I hope can be a natural framework for evolutionary theory of why naively unlikely but energy-efficient multi-cellular storms are so common." The short answer is no. Many of these words that you used do not quite fit together to match anything that is going on in my mind right now, so I'm not quite sure what you have in mind. So I don't have any specific ideas or examples along these lines.

1:09:35.7 SC: The cloud example, I should say, is very different than what we studied, because as I mentioned, the whole point of what Scott and I and our collaborators are studying is that we're looking at closed systems. If you have open systems, that is to say systems that are driven by forces from the outside, systems of interact with the rest of the world, then you can sustain complexity for a long time. You can have turbulent systems, the great red spot on Jupiter, not to mention all the rest of the fun bits of complexity in Jupiter's atmosphere, these are driven by dynamical processes, and energy is being pumped into them.

1:10:16.5 SC: And in statistical mechanics, we have what are called non-equilibrium steady states. And that is something that we don't know a lot about them, but they are known to exist, and people study them, and so forth. There's several famous examples of things like these playing important roles. And so clouds would be one example, clouds are not closed systems, clouds very much interact with the world around them. So if you just kept pumping in the right kind of energy, a cloud could exist forever. Whereas in what Scott and I were studying, you are tending toward equilibrium. Since you're not pumping in anything from the outside world, you will eventually... That cream, that coffee would just reach equilibrium and then stop. There's nothing more macroscopically that's going to happen. So we're looking for a self-generated complexity, not complexity that is parasitic on energy being pushed in from outside.

1:11:13.9 SC: Liam McCarty says, "Why is there so little scatter in the Baryonic Tully-Fisher relation? I saw you say in a recent interview with Brian Greene that there's overwhelming evidence for dark matter, but I struggle to understand why there would be so little scatter in the dark matter paradigm. And I've been struck by the fact that MOND predicts exactly a relation that matches the data. I know MOND fails for galaxy cluster dynamics and more, but I'm curious to hear your thoughts on why it predicts as much as it does despite being incorrect?" Well, so this is, again, an expert level question. Let me try to back up and fill in. The Tully-Fisher relation is relationship between the mass or the density, I guess, I forget, of a spiral galaxy and the velocity that it has, the asymptotic rotation velocity. And this is a interesting, important astrophysical empirical relationship.

1:12:11.0 SC: It's a little bit different for spiral galaxies and elliptical galaxies, et cetera, but nevertheless, it is there. I don't think that the scatter is very small, I think that there's pretty noticeable scatter in the Tully-Fisher relation. But it's an interesting case, I'm answering the question not because I have an interesting answer to it, but because it raises interesting questions within this field. There are people who advocate against dark matter and in favor of modified gravity. MOND, the theory performed by Mordehai Milgrom is a favorite example of an alternative to dark matter and MOND makes certain very specific predictions, and the Tully-Fisher relation is one of them, it explains that quite well. Whereas in the cold dark matter paradigm, the Tully-Fisher relation, and other... There are other features of phenomenological galaxy structure and so forth that are not obviously predicted by the cold dark matter paradigm.

1:13:19.3 SC: But the point is it is completely possible that we just haven't predicted them yet, the relations that become important in these galaxies that MOND purports to explain are always in the regime where the dark matter by itself is not doing the work, it's exactly where the dark matter and the ordinary Baryonic matter are sort of both important at the same time. And Baryonic matter, despite the fact that it is what we know about from experiments and observations and so forth, is actually much harder to dynamically model than dark matter because dark matter is just collisionless, and it's just feeling gravity, that's it. The Baryonic matter feels magnetic fields and just light from stars and supernova explosions, and there's all sorts of interactions, and it's very, very complicated and no one claims to know exactly what's going on.

1:14:15.1 SC: So the fact that there are features in galaxies that are compatible with the cold dark matter theory, but not directly predicted by it, to me is the least surprising thing in the world. I would just expect that to be the case given how complicated astrophysics and magnetohydrodynamics are, whereas, as you say in the question, MOND fails for galaxy cluster dynamics, it fails for the cosmic microwave background. It's not that it doesn't quite make a prediction, it makes a prediction and the prediction is wrong, so I honestly struggle to see why anyone sees similarity here or a kind of equivalence. One theory has failed, the other theory has not yet answered every question you can ask about it, those are not equally big problems. And when the theory fails, you should move on. When the theory doesn't yet answer all the questions you can ask of it, you work harder to answer those questions.

1:15:22.4 SC: So I think that astro-physically by all means put work into understanding the rotation curves of spiral galaxies and the Tully-Fisher relation and all these things, but to think that they are evidence against dark matter just seems a little weird to me, to be honest. Henry Jacobs says, "In your recent interview with Brendan Nyhan, he expressed skepticism about the impact of filter bubbles, did this alter your perception on the issue?" Not that much, but it was priced in for me already. I read Brendan Nyhan's stuff and other people's stuff, and I think that it's another example of life being complicated. Okay? At some point people started saying, "Oh, now that we have social media, you can't have filter bubbles, you could only be exposed to opinions you like." But it was always gonna be more complicated than that and guess what? It is more complicated than that.

1:16:18.8 SC: Zeynep Tufekci expressed similar thoughts years ago, when she was on the podcast. And someone else did, I'm forgetting who, maybe not on the podcast, maybe something I read somewhere else, but the point is most people are in fact exposed to all sorts of different opinions, they just don't listen. Or maybe it was Hugo Mercier, actually. People do see opinions and facts that go against their particular belief systems, but they're really good at ignoring them. That's the problem more than the filter bubble problem. So I think that I'm on Brendan's side with that, but I don't think that it was... It was not that new to me, the details of course are always interesting and useful, when you talk to someone who really knows what they're talking about. Jason [1:17:09.3] ____ says, "In your podcast with Yejin Choi, you stated when people say alignment in the context of AI, they mean aligning the values of human beings with the values of AI."

1:17:20.0 SC: "Which sounds like a good thing to do, but then again, I'm not sure that AI has values. I worry there's a category mistake going on here." Choi responded, 'Humans have diverse values depending on different cultures, but we also just have personal choice, so I believe in value pluralism, where we just have to respect a lot of different values. And the question is, what does that even mean, to align, to diverse values?' I agree value pluralism would be the right approach to AI alignment, but I'm wondering the answer to the question in her response, if you want to take a stab." Well, I'm not sure I wanna take a stab, actually, because the point I was making was a little bit different. I probably should have followed up on this better than I did in real-time. To say human beings have diverse values, sure, yes, I completely agree with that. That's fine.

1:18:09.8 SC: And therefore, that is a challenge when aligning AI's values with our values. But my question was, does AI even have values? It has something, it does something, but we are being anthropomorphic when we talk about AI as a thing that has values that the... Or at least in principle, it is not completely clear to me that the concept of a value is something that makes sense for an AI to have. Maybe it does or maybe it will someday or something like that, but we shouldn't just talk that talk without giving serious thought to whether or not the words we're saying make any sense. The question to me is not, to who which values do we align? It is does it even make sense to talk about the values that an AI would have in order to align them with whatever we might want them to be? I actually think that it won't be that hard if the AIs do have values or there comes to be some pragmatic understanding of what it means to talk about AI having values to give them values.

1:19:25.9 SC: Even though different human beings do treat each other differently, do have different ideas about what the values should be, et cetera, that doesn't seem to be actually to me the main stumbling block. We can train the AIs to be kind of blindly nice to each other and nice to human beings, and that's fine, if we can teach them anything at all, but I think that we're in danger of making a huge mistake of taking the word values that we use in the context of human beings and blindly extending them to cover artificial intelligences. I don't think that we know... Or for them there's just not anything special about the word values. I think you can say the same thing about the word knowledge, for example. The AIs we've created in neural networks, machine learning, deep learning models, think in a very different way as far as we can tell than human beings think.

1:20:24.3 SC: So we have trained them and designed them and built them to sound like human beings, therefore, we tend to gloss over the fact that their underlying thinking processes are wildly different than ours, so that's why I would want to be at least very, very cautious about using language that takes human qualities that we attribute to our thinking, in our values and so forth, and mindlessly apply them to AIs. Johann Faulk says, "Priority question. Can a pair of particles be in a superposition of entangled and not entangled? And are there any ways of doing a Bell's inequality-like test to test this?" Well, sure, no problem. You could have two particles that are in a state of being entangled, and you can have two particles, the same two particles could be in a state of not being entangled and you can superpose them, that's what quantum mechanics is all about. Quantum states can always be added together to make a superposition.

1:21:24.8 SC: The way that we would talk about it, if we actually had that, is not to say that the particles are in a superposition of entangled or non-entangled, rather we would just say they are not maximally entangled. Entanglement is not a yes or no question, there's a degree of entanglement. So if you think about two spins, for example, and the usual example that people have is the spins are anti-aligned with each other, so spin-up and spin-down, plus, spin-down and spin-up, that would be an entangled state. What would be an un-entangled state? Well, if you knew the first one was spin-up and you knew the second one was spin-down, that would be un-entangled because you know both of them, they're both completely known, and there's no nothing new that you learn about the second spin by measuring the first. So take the entangled state, which is spin-up and spin-down, plus spin-down and spin-up, and just add a little bit, add Epsilon, spin-up, spin-down.

1:22:28.1 SC: So then you get unequal contributions from spin-up, spin-down, and spin-down, spin-up, and suddenly you have the superposition of entangled and non-entangled, it's just that you've lowered the entanglement a little bit. Are there ways to test it? It's always a little bit complicated, but the proper answer is going to be yes. If you know anything about Bell's inequality, you know that you can't test it with just a single measurement, it's an inequality that is statistical in nature, so you have to do a whole bunch of tests, but you could do exactly the same thing for whatever quantum state you want it. Natalie Lines says, "Can you explain a bit about the emergence of spacetime from quantum mechanics in your paper Space from Hilbert Space? I just serendipitously came across it and I'm fascinated by the concept." Yeah, so I'm answering this question for two reasons, 'cause I've talked about this a lot.

1:23:21.6 SC: I did a solo podcast, if you're interested, Natalie, Gravity from Quantum Mechanics or something like that, and there I go into great detail about it. But I wanted to answer it both because I love it when people serendipitously come across research papers that I've written and are curious, but also because I realized that at the philosophy level, I am not yet doing the best possible job of explaining what's going on. And I don't mean that in terms of my rhetoric or speaking or writing about it, I mean my understanding of it. We're talking about emergence, and I talk about emergence a lot, it's absolutely central to how I think about the world. I talk about it a lot in the Big Picture. But there are some nagging philosophical worries about how well we understand the basic concept of emergence, so I'm gonna apologize here because I forget which questions I am including here in the AMA and which I had to delete for reasons of time and space.

1:24:24.1 SC: But one thing to mention about emergence is the word... One of the reasons why the word is not very good is because it gives you the impression of a process happening over time. When you emerge from the pool and dry yourself off, you were in the pool and then you emerged from it, that is not the idea of emergence that we have here. The real idea we have at the base level is a match between two different kinds of patterns. Okay? The example I'm always having in the back of my mind is the emergence of gases and fluids from atoms. So we have atoms and the gas in the room that I'm speaking to you from right now, that I can speak of as a gas, as the air in the room. It has a temperature, it has a pressure, it has all that stuff, or I can speak of it in terms of the actual atoms and molecules of air. And so there's a parallel set of different vocabularies, a vocabulary of atoms with positions and velocities.

1:25:31.5 SC: A vocabulary of a fluid with pressure and density and things like that, and there's a relationship between these different vocabularies. That's what emergence is to me it's a relationship, not an arbitrary relationship, a relationship that preserves the relevant features. So for example, I know that if I have a certain temperature of the air in the room and a certain density and things like that, if I just leave the room to its own devices, one can predict how the temperature and density will evolve over time. And I could make that prediction either at the level of all the individual atoms and molecules or at the level of the gas. But there are some hanging questions about exactly how that happens, is it unique? Et cetera. When does it happen? And so forth. So I need to do better at my way of explaining what I think of as emergence.

1:26:19.5 SC: But the rough idea is exactly that kind of parallelism, but now between a quantum state and space, or space as in part of spacetime. So in general relativity, space has a geometry, it has as a dimensionality, it's three dimensions in the world that we know, and it has a geometry, at least if you take a slice of spacetime to make space, it will have some induced geometry from the shape of that slice. And the features like the geometry and how it changes over time, we are saying match certain features of the underlying quantum state. You can extract from the underlying quantum state the same patterns of behavior, both description at any one moment, and changing the description over time that you would have if you just had a straightforward spacetime description. That's what we mean by emergence, you can locate the pattern...

1:27:18.7 SC: I'm using pattern because Dan Dennett famously wrote a paper called Real Patterns where he explains this rough kind of philosophy, although again, I still think that there's more work to be done, but I think that you got the basic idea right. There's a matching of these patterns, that's what you mean by emergence, that's what we're looking for happening in the quantum state and physical space. And the way it happens is with entanglement, different parts of Hilbert space are entangled with each other, and they have the right kind of entanglement, and the right pattern of entanglement to match what we know is supposed to happen from good old space and good old quantum field theory. It hasn't completely happened yet, we put a lot of it together, but there's still work to be done there, so let's see how it goes.

1:28:03.2 SC: James Tyler Can says, "From what I understand, in quantum field theory, every particle is really just an excitation within its field, and there's a field for each type of particle. However, I've also heard that at high enough energy levels, the electromagnetic forces, the weak nuclear force and the strong nuclear force merge into one force. Does this mean that the fields that represent these forces actually merge into a single field, or is it that the excitations in the fields become indistinguishable? Or is it something else entirely?" I think it's closest to the last one, the excitations in the fields become indistinguishable. There's a little bit of vocabulary issue here, as there often is in these situations, when we talk about how many fields we have. So for example, in good old, strong interactions, I don't know if that's good old for you or not.

1:28:54.9 SC: But the strong interactions are something that particle physicists understand pretty well. You may have heard that there are eight different kinds of gluons in the strong interactions, so the question arises... And that's true. So then the question arises, do we have a single gluon field that can vibrate in eight different ways, or do we have eight different gluon fields? To be super clear, nobody cares about the answer to this question, you just have to figure out what way you're gonna talk ahead of time. And the real point is that in these theories called grand unified theories, which are entirely speculative right now, we don't know whether they're true or not, in which electromagnetism, weak nuclear force and the strong nuclear force merge into one force at very high energies, they would be like the eight different gluons, except there'd be more of them.

1:29:48.9 SC: There'd be a bunch of different gauge bosons, all of which were similar to each other, all of which were basically indistinguishable from each other, which you could call one field, one grand unified gauge field, or you could call it a set of different fields that are all similar to each other and could be traded off for each other. It doesn't matter what language you use. What happens when the grand unified symmetry breaks in these theories, which may or may not be true, is that those different parts of the single grand unified field start behaving differently. Some of them get masses, some of them a couple to quarks, some of them don't, and they become what we now call the strong interacting gluons, the weakly interacting W and Z bosons, the electromagnetic photon. There's an intermediate step, of course, because before you split off into E and M and the weak force, those gauge bosons are unified together.

1:30:51.9 SC: That's a more complicated story 'cause they're not super duper unified, but that's basically the idea. At super high energies, if these theories are right, all of those gauge bosons look exactly the same, and they only differentiate once you have that symmetry breaking. Matt Rogers asks a priority question. "In his book, Review of Incomplete Nature, Dan Dennett said that Terrence Deacon's theory of emergence, and specifically how mind emerges from matter is the best theory on the topic he's ever read. Since emergence is an area that you care about, I was wondering if you are familiar with Terrence Deacon's theory of emergence. If so, then what do you think about it? Would you be willing to do a podcast interview with him as a guest, so we can hear you agree or disagree with him in public?" So I know Terry Deacon a little bit, he was a participant in the workshop that I organized, Moving Naturalism Forward.

1:31:41.3 SC: You can find his contributions there on YouTube, we have all the YouTube videos there. And I think he's a very smart guy doing very interesting things that are closely related to things that I care about, but I'm not an expert on what he says. Look, this is the dangerous thing about asking priority questions, usually if you ask a question, "Are you familiar with X?" And I don't answer it, it's because my answer is "No, I am not familiar with X." So I'm just not familiar enough with his theory to say anything very much about it. As far as having him on the podcast goes, I have a very strict policy of never commenting pro or con about having people on the podcast because there's a whole bunch of reasons why they might not want to come on to the podcast, maybe I've already invited them and they're gonna appear in the future, or maybe I have invited them and they've said no, or maybe I have some reason to think they would be a bad guest, or maybe they're just too busy, who knows?

1:32:43.7 SC: But I think it's unfair to my guests, my actual guests or prospective guests, to talk about their suitability as guests before they actually ever appear on the podcast. Paul Conte says, "Why is the topic of climate change so politically polarized in the USA? We also have conservative political parties over here in Europe, the present governing party in the UK is actually called the Conservative Party. But all these parties accept the scientific basis of anthropogenic climate change, yet in the US, it seems to be split along Republican and Democrat party lines, which seems most unhelpful." I think it is most unhelpful, that is absolutely true. And I think it's a sort of singular pathology of the United States, and you have to understand, when the US is doing one thing and the rest of the world is doing the other thing, don't try to understand the rest of the world better, try to understand what's weird about the United States.

1:33:38.3 SC: And there are many things that are weird about the United States, and very often; this is gonna sound very strange 'cause the question's about climate change, very often, it comes down to race and slavery and the history of the United States, most particularly the Civil War. The Civil War was the unique event that really made the United States just different than everywhere else, and what I mean by that... This would be a topic for... And I'm not a super expert here, so feel free to disagree if you want. This will be a topic for a much longer discussion with a real expert. But we had this thing in the United States where there was a split between the South and North, and there was an economic split. In the early days, the North was more industrial, the South was more agricultural, et cetera, and of course, unmistakably, the South had slavery, and all those facts kind of mixed up with each other.

1:34:32.6 SC: The slavery, it was not coincidental that the part of the country that had slavery was also the agricultural part, et cetera, and for various reasons, it became the poor part. If you look at which states have the highest per capita income, which states have the best universities and things like that, it is... If you go back to the states that were there before the Civil War, it's the northern states, not the southern states. And so there was this weird mixture of poor people in the South being resentful of the North because of the Civil War and things like that, a perception that they were looked down upon that was very cultural-based, not particularly class-based or income-based, that absolutely drives a lot of American politics. So there are... The point is, this gets into identifications with political groups that have salience in the United States that have no analog in other countries.

1:35:46.4 SC: So the Republican Party in the United States is this bizarre coalition of super-wealthy business people and super-poor people who are less educated than average, more driven by resentment than average, more racist than the average. Again, all of these are on average, they're shining counter-examples, of course. But if you tell people in the United States, the average Trump voter, the median Trump... I don't know if it's the mean or the median, to be honest, so I shouldn't say. But in a very sensible way of slicing it, the average Trump voter had a higher income than the average Biden voter, for example, or a Hillary Clinton voter, they will be surprised because the stereotypical Trump voter is poor and uneducated and whatever, but it's this weird coalition that we have. And there's some of that in other countries too, but it's much more pronounced here in the United States than elsewhere.

1:36:46.0 SC: And so what this has to do with climate change is there are a bunch of kind of cultural levers that are important in the United States that the game theory, politicians can figure out, I can take this issue and I can excite my base with it, because they feel like the world's against them, and they get very resentful about it and so forth. There are very, very wealthy people who own businesses and so forth, who don't want political action taken to ameliorate climate change because it would cut into their bottom line. And there are very, very poor people who think that the liberal elites from the coasts in the North and the universities are trying to take away their way of life. And these people are sort of in class terms on opposite sides, but they are in the same political party in a way that is just more obvious in the United States than anywhere else.

1:37:49.1 SC: So there's no logical sensible reason for people to be against fixing climate change any more than there is sensible reasons for people to be against vaccinations or anything like that, but they go together in a sort of cultural identity sense that leads to some bizarre, kind of bizarre bedfellows and kind of bizarre outcomes. And it's much more complicated than that, but I'm just trying to help out if you're not a United States citizen try to figure out why the United States is so weird in so many ways. This also helps to explain why health care is so bizarre here in the United States, various other things go back to our very weird kind of unique history. Christian Hoffman says, "I would like to know what's the set of things that quantum fluctuations can produce. Is it true that at every moment in time and for every possible matter distribution in space, there is a branch of the wave function, however small in weight that contains that distribution? I believe you did say the charge is conserved in all branches, so that would put at least one constraint on the matter content."

1:38:54.5 SC: I think this is a complicated question to be super precise about, but the overall impression one should have is not that hard to get, namely, no, it is not true that quantum fluctuations can just produce anything. Quantum fluctuations produce what the Schrodinger equation allows for, that in many worlds or in quantum mechanics more generally, et cetera, et cetera, et cetera, and no matter what version of quantum mechanics you like, the Schrodinger equation tells you what's gonna happen at least with certain probabilities. And it never says everything will happen, it says certain things will happen with some probabilities, the probabilities might be zero. For example, the probability of any transition where the total electric charge changes is exactly zero, but there might be other cases where the probability is exactly zero. So even as, there might be a whole lot of cases where the probability is small, but not quite zero.

1:39:55.5 SC: I think that the intuition you should have is very much not, "Anything could happen." It very much is, "Certain things happen and certain things don't." It's what the Schrodinger equation allows for, that matters. Aumie says, "If we were to ever find a theory that would unify GR and QFT, do you think the equation for it would be something short and elegant, like the energy-mass equivalence, or do you think it would fill up pages with lengthy if then, else-type scenarios?" it's a reasonable question. I think it would be short and elegant. It's not a perfectly well-defined question because as many people have pointed out overall, you can always come up with clever notation so that even the most complicated equation becomes simple. You have some very, very complicated equation, you move all the terms to the left-hand side and you call it X, and then on the right-hand side, you have zero, and so the equation is X equal zero, all of the details are hidden in the definition of X.

1:40:55.2 SC: And you could do likewise even if it's not a single equation, even if it is then a list of if then, else-type scenarios, but that's cheating, I think we kind of all know that is cheating. I think that there's... I don't know if there is a way to phrase this question in a better defined way, to be honest, but the spirit of it is, is there's kind of a unified thing, that the world is in, a unified way that that thing behaves. And maybe it shows up in different ways, but is there an underlying single, elegant behavior for this stuff. In my Naturalism class that I'm teaching right now, we're just reading Nancy Cartwright, who is a philosopher of science, who very much believes the opposite. She defends an idea of a patchwork physics where you're just gonna need different language and different concepts to discuss different kinds of physics, different kinds of physical stuff.

1:42:00.0 SC: I just don't see any motivation for that, honestly, I think that the entire history of physics has been moved towards simplicity and unification, there's no... On the other hand, there's no principled argument that it has to keep going that way, so it's absolutely possible that the final theory would be a complete mess, but if I were to bet, I would bet on it being pretty simple. Chris V says, "Could you briefly talk about The Feynman Lectures on Physics? If I wanted to build toward a working knowledge base from forces to quantum field theory in my free time, would you recommend them as a good place to start?" No. No, I would not. The final lectures are great once you've already learned physics because they're extraordinarily insightful, they use clever pedagogical techniques and things like that, but they're a little quirky and they're quite high-level.

1:42:51.8 SC: So it would be very, very difficult, I think for most people to learn physics for the first time starting from the Feynman lectures. And also they don't go that far, they were very explicitly aimed at the first three semesters of an undergraduate physics majors education, so they don't do quantum field theory, they don't even purport to try to do quantum field theory or general relativity or anything like that. The books I've just written, try to do exactly that, so you could try those, but it's not supposed to be a working understanding in the sense of solving problems, if that's what you mean by working. You're supposed to understand the concepts from reading my books. But I would absolutely recommend having them around The Feynman Lectures, if you were learning from something else and wanted an extra reference to compare to.

1:43:43.5 SC: Melita S says, "I'm curious as to how you process reality. How does this globe make you feel? That all we can retrieve from reality is that we share entropy, aka, life cycle with our loved ones. Does it generate fear, confusion, anger, nothing? It feels like until we find the theory of quantum gravity and going beyond quantum field theory, something big is missing from our understanding of reality." Well, there's a lot going on here. Let me try to at least comment on it. So you say all we can retrieve from reality is that we share entropy, aka, life cycle with our loved ones. So number one, I'm not sure what is meant by sharing entropy with our loved ones, both we and our loved ones increase the entropy of the universe. And I also don't know what is meant to say entropy, aka, life cycle, that I'm not quite sure how that connection is made.

1:44:38.5 SC: This is like in the vicinity of something that I think is very important and fascinating and deserve to be better understood than it is about how in the process of increasing entropy, human beings do things like form memories and communicate and share information. All of this is highly tied up with entropy increasing, but it isn't simply reduced to entropy increasing. And then you're asking, does these facts generate, fear, confusion, anger, or nothing? Yeah, I would say nothing. I would say maybe it generates intellectual curiosity, I want to understand how it works, but if you read the big picture, I try to be very explicit here that we should understand the world at different levels. The levels need to be compatible with each other, you can't have one level of understanding that is just in conflict or in contradiction with another level. But every level is autonomous, you can talk about the world at that level.

1:45:38.6 SC: So when I talk about human beings, my loved ones, et cetera, I'm just not thinking about entropy and atoms and stuff like that, those are not the relevant concepts to use. Finally, you say, until we have a theory of quantum gravity, something big is missing in our understanding of reality. Yeah, sure. Quantum gravity is missing, that counts, says something big, but again, it's not going to affect our human level of comprehension, there is nothing big in fundamental physics that is gonna change who we are as human beings. Leyland Belmont says, "I want to better understand emergence." Me too, Leyland, so we're on the same page here. "Is the claim life emerged from a chemical soup, any more rigorous, insightful and formative specific explanatory or scientific, than the claim, there were chemicals and then a miracle happened, and now there is life?"

1:46:30.6 SC: So first thing is, that's not emergence, that is exactly the misunderstanding of the word of emergence I was warning about a couple of questions before. When we say life emerged from a chemical soup, that is not the idea of emergence that we have in mind when we're talking about the emergence of one higher-level description from a lower-level description. The life coming from chemical soup is supposed to be a specific physical process playing out over time. In terms of whether or not that suggestion is more rigorous insightful, et cetera, yes, absolutely, it is much more rigorous than a miracle happened. Why? Because we can study it, because we can figure out what the process was, because we know what the basic ingredients are, we know what the underlying laws are, so we can do science to it, whereas we cannot do much science to the claim that a miracle happened and now there is life, that's a much less helpful perspective.

1:47:28.9 SC: Eric Daviji says, "We've heard of the technological singularity, but do you think there's such a thing as a cultural singularity or even just a hard cap on how complex human culture can become as we accumulate more and more cultural artifacts and recorded history?" No, but I don't think there's a technological singularity either...

1:47:49.0 SC: Let's back up, it depends on what you really mean by a hard cap, there is, of course, a cosmological upper bound to how long civilizations and people can survive, how much entropy and information we can talk about, but we are so, so, so, so far away from that, that in no practical terms, is that at all relevant. And I think that the talk of the technological singularity is entirely silly based on plotting things that have no numerical values to them and then scaring yourself into thinking that something's going to happen.

1:48:26.2 SC: There are phase transitions, there are moments, there are tipping points where things happen, where things change dramatically and irrevocably, and something like that could happen. But to think of it as a hard cap is I think the wrong way of thinking about what is the future of human culture. Physics Kitten says, I know that relativity treats gravity as the curvature of space time, I understand how that works, and examples like the orbit of planets around the sun, but how do I make sense of a curvature of spacetime interpretation in the case of tides, which I think of is being caused by gravitational attraction of the oceans on the side of the earth, near the moon.

1:49:05.7 SC: What space time is curved, and is there a relativistic way to describe what is going on? Yeah. This is a 100% a relativistic way to describe what is going on, and I'm not quite sure where the confusion arises here because it's kinda clear in my mind, so I apologize if my answer isn't helpful to you, but the space time that is curved is the this space time through which the earth and its oceans are moving.

1:49:30.6 SC: The reason why it is curved is because the moon and the sun to some extent are curving it, and that curvature shows up as what we would have thought of pre-relativity as the force due to gravity, and it's exactly the same. There's no difference in the solar system here in on earth anyway, between the effective force of gravity that Newton would have predicted and the effective force of gravity, the general relativity predicted. It's just that the explanation for it is different, the curvature of space time... Space time is curved in a very particular way that matches on exactly to the Newtonian gravitational potential, so the workings of things like tides are no different whatsoever in general relativity than in Newtonian gravity. There's other things that can happen like gravitational waves, the Newtonian gravity doesn't have room for, but the good old Newtonian phenomena are a subset of all the possible things that general relativity accounts for.

1:50:34.2 SC: Tomash Gashdos, says can you talk about where the idea of dark energy comes from and what evidence supports it. You talked about the evidence for existence and properties of dark matter in your recent solo episode but dark energy wasn't covered in that much detail, and I'd love to learn more about that too. Kind of there's less to say about dark energy in two senses, one is unlike dark matter, where we have many different theories that could be right, but we just don't know which one is right. For Dark Energy, we basically have one really, really, really good theory and a bunch of less good theories, the one really good theories are that the dark energy is the cosmological constant, it is just an energy in the vacuum, an energy that is inherent in empty space itself, it's the same amount of energy in every cubic centimeter of space.

1:51:26.0 SC: An idea, going back to Einstein, we talk about it in slightly different ways in the modern world, but it basically is that idea, and you can pause at this as Einstein did, he later gave up on it, but he didn't completely forget about it. It was always lurking in the back of the minds of cosmologists as they tried to confront various observations that came on, finally, in 1998, we found very good evidence that the universe is accelerating. That is exactly what you would expect if about 70% of the energy density of the universe where this dark energy stuff, this vacuum energy stuff. The problem is no one really expected it, people had absolutely been thinking about it, but no one would have bet you 50/50, that it was there. So we were surprised, and the reason why we were surprised is because it seems very, very unnatural that you would have exactly that amount of dark energy rather than some other amount.

1:52:20.8 SC: That's a longer conversation I don't wanna get into right now. But because we were wrong about the expectations there, we are trying to be cautious and people said, Well, even though we have a perfectly good theory that fits all the data the cosmological constant we should still be cautious, so we should allow for something that is dynamical, something that is not completely constant, and we do that and we try to test it, but guess what, all the tests are saying that if there is any dynamics to it, it's very hard to notice practically zero, just what you would expect if it's not dynamical at all, so people like me had proposed modifying gravity to get rid of the dark energy, but it doesn't fit as well as the original vacuum energy idea, so the smart money is it's just vacuum energy, it's just the cosmological constant, if it's not, there will be some data that will come in and will change our minds.

1:53:21.7 SC: Stephan Burnicker says, how is entropy defines an open system like the universe where total energy within an expanding volume is increasing and where gravity is ordering matter. Well, it doesn't matter. None of those things matter. As long as you have some distribution of particles, for example, once you have fields, it does become trickier because fields can do an infinite number of things very easily, but let's stick with just particles, then you have a distribution function, you have a probability that there will be a particle with a certain position and a certain velocity, call this F. Okay. The distribution function, the probability, F is a function of X and P, the positions and the momenta of the particles. Then there's an equation, F equals minus the Integral of F log f. S sorry, entropy equals minus the Integral of F log f.

1:54:13.2 SC: That's it. It's basically a way of saying how tightly constrained, that probability distribution is, a probability distribution that is very, very narrowly concentrated in some region is said to have low entropy, and one that is spread out all over face base is said to have high entropy. So this idea of entropy that I just told you about arose from a phenomenological thermodynamic understanding from the 1850s or whatever, before we knew about atoms and molecules and so forth, and then the definition of entropy was something that only applied to equilibrium situations, but now since the 1870s, honestly, we can do better than that, there's no reason to assume equilibrium.

1:55:00.3 SC: Sean virtue says, how do we collide such small particles in accelerators, I can hardly throw a baseball in a straight line, I can't imagine the precision needed to collide two protons moving at 0.99 C. They moved a lot faster than 0.99 C, at least relatively speaking, 0.9999999999 C, something like that. But I get the question, and the answer is roughly speaking, you just throw a lot of particles at each other, This is what is called the luminosity in a particle accelerator. So you have a beam of protons, for example, and you try to make the beam as thin and as focused as possible, so the density of particles is as high as you can get, and then you zoom them around at high velocities, you zoom... Sorry, you zoom a huge number of particles around a ring in your particle accelerator very, very fast and push two of those beams of particles at opposite directions in the middle of a particle detector, most of them do indeed just go right by each other, but sometimes you see, a spectacular collision. It just work out the numbers, 'cause there's a lot of them there. And by the way, it's tricky, right. Think about it?

1:56:15.5 SC: I'm trying to have a beam of particles, it's not a continuous beam, it comes in bunches because I need to turn magnets on and off really quickly to give them a little nudge, just to give them a little acceleration, which means that there needs to be like a discrete size of protons that can go through the magnet and be pushed at the right time, a smooth continuous stream of them would not work, and the other thing is all the protons are positively charged, so they are repelling each other the whole time. They want to spread out. So we need other magnets to focus them really, really tightly, and this is an enormous technological achievement that we can do that. It's something like the LHC.

1:57:03.9 SC: Aaron Bouton says, listening to your Physics in crisis solo edition, you mentioned the use of cut-offs in effective field theory to avoid considering ultraviolet energy levels in the equations, can the same trick be applied to black holes and the Big Bang to avoid the singularity and relativity. Or am I asking a mathematically naive question? It's not a naive question, but the answer kind of is no, sorry about that.

1:57:26.3 SC: At least as far as anyone knows, the kinds of infinities that are coming up in relativity at the singularity are a different kind. There a physical quantity, mainly the curvature of space time going to infinity at a physical place in space time, the singularity, that's a different thing than what you have in quantum field theory, where you kind of get a fake infinity from virtual particles, and virtual particles aren't real particles, that's why they're called Virtual particles, they are a way of doing a mathematical calculation when the fact that they give you infinity it's just an indication that they're a bad way in some cases, and the ultraviolet cutoff is a way of admitting that these virtual particles with arbitrarily high energies weren't really necessary in the first place, whereas as far as we know, inside the black hole that Singularity is necessary, at least as far as classical general relativity is concerned. Sumir says, could you please explain how it is possible to prepare a qubit in a specific state and then subsequently put it through a quantum gate resulting in the qubit now being in a superposition, e.g: A Hadamard Gate, Hadamard Gate is one of the various logic gates that we use in quantum computing.

1:58:42.2 SC: So I'm not sure what you'r hoping... What you're thinking is the difficulty here, the original preparation or the later evolution to prepare a qubit... By the way, I should back up. Actually. A qubit is not a thing, there are various kinds of things that could represent qubits, a qubit is a two-state quantum system, such as the spin of an electron or something like that, but there are other things that could be a qubits. Let's just call it a qubit, realizing there's different physical manifestations of these things, so if it's an electron with a spin, for example, you just measure the spin and you get either up or down.

1:59:19.9 SC: And therefore, as soon as you measure it, now you know exactly what it's in before you measured it, it might have been in some superposition now, you know, so that part is relatively easy to put it in a specific state by a quantum gate, you just rotate it or something like that, you put the electron in a magnetic field and you rotate it a little bit, so if you are measuring spins or discussing spins relative to the Z-axis, if you rotate it from being exactly spin up along the Z-axis to being halfway in between spin up and spin down, then it's spin plus X. It's a perpendicular angle, and that corresponds to a superposition of spin up and spin down, so you don't have to untangle it to actually manipulate it, that's part of the joy of things, entanglement happens in a specific kind of interaction, one that would interact differently with the two different states that are in superposition, but just rotating the qubit or something like that, the magnetic field doesn't count as entanglement. That's something you can just do to it.

2:00:30.3 SC: So you can even do that if you don't know what the original state is, I can rotate the angle of its axis by 90 degrees without knowing what that original axis is, that's an important thing for a logic gate, important property for it to have. Brandon Hall says, I'm having a very college student moment, and I've been moving towards socialist ideas and Marxist critiques of society, and I realized that though you had a great discussion on your moral and political opinions about animals and IDW things. I don't believe I've heard your comments on socialist subjects, do you accept these sorts of criticisms of capitalism and perhaps accept any of the more radical opinions of socialist traditions? I have some sympathies, but I'm not a 100%. One thing is, I just haven't sat down and decided on an entire critique of society or favorite political system or anything like that, so I'm not trying to advertise the solutions to these grand problems, I tend to think both, that capitalism is very, very good for some things, it absolutely helps to set prices and equilibrium, it helps stimulate innovations, a whole bunch of other things, and that it's very, very bad at other things, it absolutely allows for and encourages inequality and concentration of power and abuse of workers and a whole bunch of other things.

2:01:51.6 SC: I don't think it's impossible to have both of those views at the same time, I think many people do, so you don't need to be extremist pro-capitalism or anti-capitalism, but as far as socialism is concerned, so I would very much like much more of the fraction of our wealth to be spent making the lives of poor people better one way or the other. I'm a big believer in just giving them money, whether it's through basic income or something like that, but I don't know if that... Will that counts as socialism. I don't know what exact definition you're going to need to have, I don't think it's in generally a good idea. For the means of production to be owned by the state or collectively owned, I don't think it's a good idea to over-plan the economy. So many traditional aspects of Marxist or socialist systems, I'm not a big fan of. So I'm kind of a wishy-washy. Welfare State, capitalist basically. I think that capitalism is a good starting point, and then you have to very dramatically modify it to make sure that its worse instincts don't take over, which they will do unless you stop them somehow.

2:03:01.8 SC: Ranble Bessenger says, you mentioned several times that you were not a big believer in heroes, your view makes sense to me as I've been disappointed in some of my heroes over time, did you always have this view or was it shaped by disappointing experiences? I don't know if there were specific disappointing experiences, I've just been around a long time, I wanna be clear about what the attitude here is, I don't strictly mind having heroes, I just think that I would prefer to admire things that people do, if I don't know them, I can absolutely be an admiring stands towards some accomplishment that someone has without necessarily therefore thinking they are a good person, because if I don't know them personally. Then what I'm admiring is the little bit I know about them, not their entirety, their [2:03:53.9] ____ as a person, and it just seems silly to me when people act surprised you, Oh, this person was a great scientist, but they turned out to be a terrible jerk in real life.

2:04:04.1 SC: Yeah, that happens all the time. There are a great scientists who are terrible jerks, there are great scientists who are wonderful people, there are people who would make terrible scientists who are never less really good people who are really bad people, I don't imagine why there should be an any correlation here or anything like that, and that extends more generally in the sense that just because someone is smart at something or admirable at something it doesn't necessarily mean they're going to be smart or admirable at a very different kind of thing.

2:04:32.7 SC: So I would much rather just be clear about what it is about a certain person's accomplishments or abilities that I'm admiring when it comes to people that you know personally, people who you feel you know well enough that you have a rounded view of who they are, I'm very much in favor of admiring people, but that's generally not going to be the case for public figures. Tim Converse says, especially in the context of talks about academic freedom and tenure, I'm curious what you can say from your own experience about pressures to be more narrow, and to what extent tenure exempts one from them? For example, if you had tenure at Caltech, would it have been easier to say, hey, so my students are turning in more philosop... My interests are turning in to more philosophical direction, so that's what I'm gonna do now. Yay for tenure. And are there contexts where breadth counts against you? All other things being equal, like, what about a physics tenure case with four great physics journal articles versus that same case with those same four articles plus two philosophy articles.

2:05:31.3 SC: You know, this is a complicated thing. It's one of those things where I hate to be wishy-washy. But every case is different. But overall, the tendency is that, Yes, I think the tenure does precisely protect professors in those situations, so tenure also protects professors when they just wanna be lazy, when they stop wanting to do research or whatever. That's the cost of doing business here. But the important thing is, it protects them for much more admirable reasons when they wanna change their research interests or be more experimental or more daring or whatever. Now, the whole system works very strongly to prevent such people from getting tenure in the first place, I mean of course, again I always kinda say, if you're a genius, you're fine, geniuses are often able and permitted to bounce around and do a whole bunch of things, but for the rest of us, for most of us, the standards are very high and departments live in fear of hiring someone and then they do something else or do something different or stop doing things at all.

2:06:37.6 SC: So they... As I said on my blog years ago, they hire on hope and they fire on fear, so you fire on the fear that you're not going to continue to do what they want you to do, so it's absolutely true that at Caltech, I could have more dramatically changed my research interests, for example, I could probably have changed them anyway, it's not perfectly clear. At the end of the day, I chose to leave Caltech not only because my research interests were no longer fit to what I had been hired to do, but just because Caltech is not as good a fit to what I wanted to do, and so I just made a decision that it'll be doing better to start anew at a job that really encourage me to do exactly what I wanted to do and that will happier for everyone.

2:07:27.5 SC: As far as breadth counting against you. I think it can, absolutely. Again, because of that fear aspect, do you know when you're giving someone tenure, you're making a prediction about what they will do for the next 30 years and whether they... What they want to do will match up to what you as the department want them to do, and if what the person has been doing is broader than maybe from the point of view of abstract intellectual achievement, that sounds good, but from the point of view of predictability It's bad, right? If they indicate that there's many different things that they might want to do, that's a danger sign, that's a warning sign, that you're not gonna be able to predict exactly what they want to do, so for departments that are just risk-averse and want to be safe in who they give tenure to, they're gonna be much happier with someone who is not broad interest, much more predictable, much more narrow and down the middle of the road.

2:08:27.8 SC: Mihai Tudor says. Recently, I learned that the Milky Way takes about 200 million years to perform one full revolution. It's also estimated to be 13.6 billion years old, dividing these numbers, I get 68 full revolutions so far, I find this number surprisingly small to obtain the seemingly complex and symmetric structure that it currently has. Do you have any insights into this? I guess my only insight is, I'm not sure why you relate the number of revolutions that the Milky Way has made to its complex and symmetric structure. The spiral arms that you see in a galaxy like the Milky Way do not rigidly rotate with it, they are more like... This is complicated, but they are more like regions where star formation is going on, and therefore the galaxy is brighter rather than regions where there's more density or anything like that. So if you're worrying how the spiral structure forms. It can form pretty easily in one or two revolutions, I don't see why it would take even 68, much less, but need more than that.

2:09:33.8 SC: So I just think you shouldn't be surprised there're 68, the number of revolutions is more or less uncoupled to the internal dynamics of the galaxy, and the galaxy has been going on for 13.6 billion years, so plenty of time to get pretty darn intricate. Robert Wrecks and Rescue says, We know that black holes are maximum entropy in a region of space time, we also know that the second law states entropy overwhelmingly tends to go up, but has a very small chance of going down. How would this accidental rare going down in entropy look like for a black hole? This would mean the black hole could just by accident and without emitting any Hawking radiation end up being smaller just by chance.

2:10:16.2 SC: Well, no, it can't end up being smaller without emitting any Hawking radiation, the energy is still conserved here, so if it's going to get smaller, it has to emit Hawking radiation, but remember, when the black hole does emit Hawking radiation it gets smaller, so the black hole entropy is less, it's just compensated for by the entropy of the radiation, so a very simple way for the black hole to fluctuate downward in entropy is to admit lower entropy radiation. And that's actually pretty easy to imagine how that could happen. Very roughly speaking, the entropy of radiation is just the number of photons that are emitted and black holes emit usually for a large black hole, very, very, very low energy photons, but a lot of them. And you can calculate how much entropy there is in the photons once they're all omitted, it's the same order of magnitude as the original black hole, but it's a bit larger, it's a bit more number of photons.

2:11:17.3 SC: So if the black hole emitted a bunch of anomalously high energy photons, thereby giving away more of its mass in a smaller number of particles, that would represent a spontaneous jump downward in total entropy. Andrew Jewel says, I have a question about dark energy in the long-term expansion of the universe, I've heard some people say that the galaxies will hold together, just keep getting further apart until each galaxy is alone in its observable universe. But Neil deGrasse Tyson asserts the dark energy will affect ever decreasing scales, pulling apart galaxies and then planets and then atoms, which of these characterizations is correct.

2:11:58.5 SC: Well, yeah, it depends on what the dark energy is, remember I said that it's probably is just vacuum energy, the cosmological constant, if that's true, then the former scenario is right, galaxies will hold together, but get further apart from each other, in fact, more specifically, gravitationally bound systems will hold together, so if you have a cluster of galaxies that is gravitationally bound, that will hold together as non-bound systems move apart from each other, that's not the end of the story in a galaxy or a cluster, the dynamics are not completely static, so the stars will bump into each other, they will fall into black holes, those black holes will radiate eventually you get nothing, nothing at all in a little observable patch of universe. If on the other hand, there's this weird and extremely unlikely phenomenon called Phantom energy, in that case, the energy density per cubic centimeter does not remain constant, but actually goes up.

2:12:58.3 SC: And in that case, which was worked out by Robert Caldwell, Marc Kamionkowski and others, then individual galaxies would be ripped apart, then there's an increasing push apart on individual objects in the universe, all the way down to atoms. But like I said, that scenario is extremely unlikely. I wrote a paper with Mark Trodden and Mark Hoffman where we explained that for that to work, you would basically need a field theory with negative energy particles, and the negative energy particles are just bad to have in your field theory. So my bet is that the dark energy is just constant and galaxies are gonna be fine until they collapse in on themselves.

2:13:41.7 SC: Jim Murphy says, I've been having an argument with a friend of mine, he believes that the general population is on the whole uninterested in philosophy and science, while I disagree, he claims that my perspective is biased by the people who I surround myself with, mainly curious people who enjoy investigating challenging questions about reality. Do you believe that the average person is drawn to scientific inquiry and deep questions, or most people blissfully ignorant? Well, look. Let's be fair to the average person, it's completely okay to not be drawn to scientific inquiry and deep questions and still not be blissfully ignorant. Okay, there's a big gap in between those two characterizations, life is short, we have to pick and choose what we're interested in, very few people are interested in scientific, deep scientific questions and deep literary questions.

2:14:34.1 SC: And the deep musical questions, and speaking lots of foreign languages and knowing how to build refrigerators and a whole bunch of other things, most things we don't get the chance to be experts on, and I think it's completely okay to not want to be an expert on too many things, there's different ways you can craft a worthwhile and rewarding life. Having said that, probably, look, I don't know because the question is a little bit too big, what does it mean to say the general population is uninterested in Philosophy and Science, maybe most people are a little bit interested, but not too interested. I would rather say that I am happily surprised all the time by the large number of people who are sincerely interested in it, I don't know what that number is compared to the majority, but there's a lot of them. I went to give a talk in Copenhagen science and cocktails, I gave a talk on quantum mechanics, and it's a Friday night and there's 1000 people in the audience just to hear a talk on quantum mechanics on a Friday night, and that does makes me very happy.

2:15:41.4 SC: The number of people who listen to Mindscape makes me very happy. So I'm not worried about the average person, I just want every person to be able to understand as much science as they are interested in. Michael Lacy says, have you made any end of life plans in particular, have you decided on a standard burial cremation, green burial or donating your body to a body farm or other research facility?

2:16:05.6 SC: Yeah, I think that no, nothing explicit, I don't have a funeral home lined up or anything like that, but cremation is my preferred thing to do and people know it, so I think that's what's gonna happen. I don't feel strongly that does the right thing to do and other people should do it, but for me, I think that's about what I want. Andre Chevanich says, is acceleration really an absolute quantity. I thought that in the framework of general relativity, acceleration is relative, that there is no experiment to find out whether one is accelerating or not, so no, that's not right. I just wanted to, it's a quick answer, but position is relative in relativity, there's no preferred position, velocity is relative. There's no preferred velocity, but acceleration is absolutely measurable, you can know whether you're accelerating, are you being pushed back in your chair or whatever you're in, there's absolutely a set of trajectories that are preferred, which are the unaccelerated or inertial trajectories in general relativity.

2:17:09.9 SC: Jeffrey Segal says, You mentioned about writing ideas in a fictional narrative. I was wondering if you ever saw the movie, My Dinner with Andre, and what you thought about it. I like this. It was quite a while ago. Well, my wife hated it. I did see it, but it was also quite a while ago for me, so I am therefore not exactly sure why you are relating this to the idea of writing ideas in a fictional narrative. In some sense, okay. My dinner with Andre as I vaguely remember, Wallace Shawn and Andre Gregory, are sitting there talking throughout the whole movie, an hour and a half or whatever it is, about slightly different approaches to life with Wallace Shawn being the slightly more down-to-earth guy. Andre Gregory being a bit more spiritual and new agey or I don't know, whether that term existed at the time. I really liked the movie. I love the movie it was fun.

2:18:01.7 SC: It's certainly kind of... I don't wanna say radical movie, but it was very explicitly... Very knowingly different, right? They literally made a movie of two people sitting down and talking, and it was not them just playing themselves. Okay. Eventhough it was close, Wallace Shawn's character's name was Wally, Andre Gregory's character name was Andre, but it wasn't like they were just shooting the breeze, it was a tightly constructed script ahead of time, they had a point for where they were going and so forth, for the right people, it's a great movie. If I did ever myself write about in a fictional vein, it wouldn't be quite that explicitly, didactic I would try to have a couple more action scenes than that one. Michael Kramer says, I'd like to ask your advice about talking to non-physicsy friends, a not uncommon question is about wave particle duality. A one-line answer I found myself using is that a reasonable way to think about it is the light and other objects travel about like a wave, but they interact with each other with other objects, like a particle. Does this seem like a reasonable way to go about this? I think it's reasonable.

2:19:16.1 SC: It's okay, but I prefer a slightly different way of saying it, which is that if you're not looking at it, this stuff behaves like a wave, whereas if you measure it, if you observe it, if you try to figure out the value of an observable quantity, what you see, looks more particle-like... Even that is not exactly true. There's subtleties there, but I think it's a slightly more sharp distinction than interacting with other objects, because after all, a particle moving in a gravitational field is interacting with the thing that is causing the gravitational field, but it doesn't cause that thing to act particle-like 'cause that doesn't count as a measurement, it's not deco hearing is not becoming entangled with anything.

2:19:58.5 SC: Sandra Stoockie says. In your reflections on the episode with Roseberry Brown, you considered two possible explanations for why your guests often end up having a background in physics, that you were not aware of when you invited them one, physics training is great and applicable in many domains and two, it's easier to come up with questions and make progress in other areas, so physicists move there, but what about unconscious familiarity bias, could it not be that some of your perspective guests have that certain je ne sais quoi, that subconsciously makes you more likely to invite them. As usual, I'm gonna confess, I don't have any idea what I said in the reflection video. The idea of the reflection videos is even less planned than usual, I just literally sit down after the podcast and record them right away, but I'm surprised I said that because I think your explanation is absolutely the right one and the one that I would give the most credence to. It's not that I randomly select people who are doing interesting things. I select people who are doing interesting things to me, that is to say. That was a wrong to say it, they're not doing things to me. They're doing things that are interesting to me.

2:21:06.6 SC: And it's therefore very unsurprising that they would be physics-oriented in one way or another, even if they end up in economics or biology or whatever. Oman Ticker says, I heard in an accelerator like Large Hadron Collider that we can't detect neutrinos, so we infer them from leftover energy from collisions, but how do we know it's just a neutrino and not some other neutral particle in addition to that. Well, basically, 'cause we know how many neutrinos there are, and we also know exactly how they interact with the known particles, so whenever you do one of these experiments, you predict how much energy should be lost in neutrinos coming out on average, and then you try to fit to the data. And if you do it to the data, then what you say is, well, that's consistent with a certain number of neutrinos, and by the way, this is why you will often see the measured number of neutrinos in this experiment is 3.1 the measure...

2:22:07.9 SC: Sorry what that means is not the actual number of neutrino particles that came out, but the number of neutrino flavors that are implied by the experiment in the real world, we have three neutrino flavors that we know about, the electron neutrino, the muon neutrino, the tau neutrino so the reason why you say something like 3.1 is because you've measured a total amount of energy that is missing and then you fit it to the prediction from knowing that there are three different flavors of neutrinos and 3.1 is very reasonable 'cause it's 3.1 plus or minus 0.2, right? And famously, The experiment they did that the best is actually not the LHC, but its predecessor LEP, the large electron-positron Collider, because electron-positron Colliders, you can measure the total energy in the initial event much more precisely, in the LHC, you say we're acquiring protons, but really inside the protons are all these quarks and gluons and those are what knock into each other.

2:23:08.4 SC: So even though you know the energy of the proton quite well, we don't actually know the precise energy of the part of the proton that did the colliding, so in a large Hadron Collider, you're not gonna have exactly the same precision for the total energy to start with, that you do at a LEPtron Collider. An electron positron Collider. So that limits on the number of neutrinos come from LEP rather than the LHC. Frederic Apollo says, I hear you say that the election on his point like, but also that it has an extent given by the content wavelength, how do we reconcile those two? So I don't think I ever say, or at least I try not to say that the electron is point like other people do, but I will say the electron is a field, not a particle, it looks particle-like, and the particle itself that it looks like has zero extent when you measure the particle, but the Compton wavelength has more to do with how well you can localize a single electron and still keep it looking particle-like... Because in quantum field theory, of course, if you put a, pump enough energy into the electron field, you can make more electrons as long as you make an equal number of positron.

2:24:21.0 SC: Right. So what happens is if you squeeze an electron to smaller and smaller distances, its energy goes up because energy is inversely proportional to wavelength in quantum theory. So the compton wavelength is the smallest wavelength that you can have for the electrons wave function, such that you can reliably say it is just one electron rather than some superposition of one and three and whatever. If you want more on that, I can recommend that you buy my upcoming book. You can race out right now and pre-order quantum fields, and that will explain these things in in a little bit more detail.

2:25:13.4 SC: Dave Gunther Geiger says, in your September AMA, you said that humanity has the power to house everyone, but we're choosing not to. I agree a hundred percent, but I don't think I know how to turn that into personal action that I can take. Do you have any ideas about how individual humans can leverage their individual power of choice to nudge collective humanity toward utopia? Well, I think that when you put it that way, you're kind of answering your own question. An individual, a single person, for most of us, for most of us individuals, has very, very, very little power to nudge the billions of people constituting the human race toward utopia. And even more than that, that doesn't mean you have zero power, but it just means that you only have a little. So don't be surprised if you don't personally lead to wide scale changes in human behavior, but you do have a little, for instance, you could vote in elections.

2:26:08.9 SC: You can convince other people to vote for the right candidates. You can donate to political parties. You can try to learn more about the issues and put your money and energy into the wisest possible schemes. And I say this very explicitly, even though it's kind of like self-evident and dopey, because I do think that there's this kind of blazayness or cynicism that that settles in that becomes a self-fulfilling prophecy when people start saying, oh, well, you know, politics is never going to fix anything. Politicians are all crooked. The parties are the same as each other. There's no point in doing it. You are the problem [chuckle] That is why politics is not very good, because people don't make the effort. I think that politics matters. I think the politics is crucially important. I don't like it when people say we shouldn't talk about politics or Politics is a distraction or politics makes you dumber or whatever.

2:27:09.1 SC: Politics is absolutely central and important. And as messy and dislikable as it may be, it's the way to have large scale change for the behavior of all human beings. So whether it's just a little bit, you know, voting or wearing a T-shirt or whatever, or whether it's diving head forth into creating political change, that's the way to make a big change in how human beings behave. Rue Phillips says, is gravity really not a force? And our physicist in denial about it. In a tweet from David Deutsch on 9 28, he said the following about gravity, no force is exerted by gravity. If you hold out your arm, the only force on it is from your own muscles. Very few people know this. I've known physicist to deny it. Sabine Hossenfelder confirmed this tweet by replying he is correct, of course, both in saying that gravity is not a force and that a significant fraction of physicists deny it.

2:28:01.7 SC: Look, [laughter], it's a definitional question. You're arguing that the definition of the word force, I can invent definitions under which gravity is a force. I can invent definitions in which it's not there. It is interesting that gravity is a feature of space time in a way that the other forces are not. That's interesting. That's at the heart of the equivalence principle and what motivated Einstein to inventional relativity and so forth. And therefore you are allowed to distinguish gravity from the other forces of nature. It's universal, right? It affects other things the same, whether no matter what they're made of, no matter how heavy they are and so forth. It's not like electromagnetism that affects you very differently if you're positively charged or negatively charged, and therefore it is perfectly legitimate to say unlike electromagnetism or the strong force or whatever. Gravity isn't a force.

2:29:04.9 SC: Gravity is a feature of space time, namely the curvature of space time. It's also completely a hundred percent legitimate to say there are four forces of nature, gravity, strong, weak, and electromagnetic forces. You know what is meant by that, and you're conveying correct information. It would be much more relevant to say, do physicists, no general relativity. That's a non-trivial question. And the answer might be yes or no. And it doesn't really come down to definitions. Most physicists, you know, most people who get PhDs in physics never take a course in general relativity. They know the popular level understanding of it, but they don't know tensor calculus or differential geometry or anything like that. Is that bad? I don't know. Most physicists don't use general relativity. I think it's much more common now than when I was in graduate school, but it's probably still not a majority.

2:29:58.2 SC: Most physicists don't need general relativity. And, but nevertheless, it is interesting, because on the outside world, you probably think that general relativity is absolutely central to a physicist education, but for most it's just not. George says, I was briefly introduced to eigenstates today in my chemistry lecture, but we were told that specific particles cannot exist at energies between eigenstates. Could you please elaborate on the exact reason why they cannot exist between these eigenstates? Well, I think your problem, George, is that you were at a chemistry lecture, rather [laughter] than a physics lecture. It's not, I mean, I get what was being said, and it's not, it is in the vicinity of something true, but it's a little bit misleading. You know, electrons are quantum particles, right? Or they're excitation in quantum fields. You have to use the quantum language to describe what is going on.

2:30:47.4 SC: So there's two things that are going on here. One is that indeed in atoms, there are energy eigenstates that correspond to the different orbitals that electrons can be in. And the general quantum state of an electron will be, even if we know that it's in a particular atom, it's going to be in a superposition of all of those different energy eigenstates. Okay? So there's no rule, it has to be in one specific eigenstate rather than another, strictly speaking. However, in the real world, if you have an atom and you put an electron in a state that is not an energy eigenstate, it's gonna decay, it's gonna give off a photon to go down to a lower energy, and in fact, it's gonna go down to the lowest energy it can. That might not be the absolutely lowest energy of all the possible eigenstates because maybe there's another electron already there.

2:31:41.5 SC: So given that there are already electrons there up to that one constraint, and the Pauli exclusion principle that says no two electrons can be in exactly the same state, the electron will go to the lowest possible eigenstate it can. Therefore, after you let the electron settle down, it is an empirical fact that all the electrons will individually be in energy eigenstates. Lewis B says, are there good theoretical reasons why we think that a dark matter candidate WIMP should weakly interact as opposed to being a non-interacting massive particle, completely non-interacting, I guess? Or is this just a case of being hopeful it interacts because otherwise finding it would be impossible? No, it actually does matter that it's weak interactions. This is called the WIMP miracle, because you can't just say, I have a particle that is dark that would make a good dark matter candidate.

2:32:34.9 SC: You have to explain why it has the abundance that it does in the universe, why there is a certain number of dark matter particles that gives you the right energy density compared to photons, for example. And what generally happens is at the very, very early universe, when the temperatures are extremely high, if the temperature is higher than the mass of your particle when E=mc^2, so we're measuring mass and energy and temperature all in the same units, then you're gonna be creating and annihilating your particle over and over again in the thermal plasma. Whereas when the energy goes below the mass of the particle, so the temperature goes below the mass, then the particles stop interacting, and then more or less they do what is called freezing out. They basically have a fixed relic abundance. So if you think about it, if you know the mass of the particle and you know its interaction strength, then you can go backwards and say, okay, when did it freeze out?

2:33:33.9 SC: And how much particles should be left in the current universe, this is called a freeze out calculation, and you get the right answer if the particles you're thinking about have approximately weak scale interaction strengths and masses. So that's nice. I mean, there's no reason why the right relic abundance of dark matter should have anything to do with the weak interaction of particle physics, but they do apparently, you know, if we were strongly interacting, if you got the, if the right relic abundance corresponded strong interactions, you might be in trouble. It might be harder to fit the astrophysical data that way. And if the, if the relic abundance that you wanted came out of a interaction cross-section that was much lower than the weak interactions, then you would have to invent a whole new force, right? Which we, you could do of course, but it's been very, very suggestive that the right abundance comes out of putting in the weak interactions.

2:34:34.0 SC: Doesn't mean it's right, but that's, a hint hopefully, that we can go help to use, to go look for the particles, which is what we're doing. OWA or OWE says, I'm working through the older podcast and just went through the conversation with Mike Brown, AKA Pluto killer. I got really curious and started reading up on the hypothetical Planet nine. One of the proposed theories is that it's not a planet, but rather a primordial black hole. Putting aside the probability, my initial thought was of concern as black holes can be pretty destructive objects. How dangerous would it be for there to be a planetary mass black hole within 500 astronomical units of the solar system? You know, roughly speaking, not dangerous at all. [laughter], for one thing, if it's in orbit, planet nine has an orbit, and that orbit is very far away.

2:35:24.5 SC: So there's no reason for it to go out of orbit unless some nefarious aliens want to, move it and point it at the earth. For another thing. It a black hole. The mass of the earth is a very tiny thing, actually. It's certainly no more dangerous than a planet, the mass of the earth. If a planet ran into the earth, that would be bad. If a black hole ran into the earth, that would also be bad. But one wouldn't necessarily be worse than the other. Maybe even in some sense, the planet is more dangerous because since it's bigger, the chances that it hits the earth are bigger. But overall, an object, whether it's a planet or a black hole, that is the mass of the earth, that's just not a lot of mass solar system speaking. So I would be much more worried about comets and asteroids, that might impact the Earth.

2:36:15.7 SC: 'cause just there's a lot more of them, even though they're much tinier. Davis Yoshida says, David Albert and Barry Lower both expressed skepticism about the idea of self locating probabilities. They both said something along the lines of needing to see more details about how these probabilities behave. Now that it's been a few years, do you feel their concerns have been answered? If so, where can I read more about the topic? Well, I don't know if this is useful or not, but I thought their concerns were already answered. I don't think there's any new thing that has happened in the past few years that clears anything up. But I think that even when I was talking to each of them, my answers were perfectly satisfying to me. Depends on what you mean by behaving like how they behave. They behave exactly like probabilities.

2:37:00.2 SC: These self-locating uncertainties are attached credences. If you have an exclusive set of possibilities, the credences for all those possibilities will add up to one. There will be numbers between zero and one that add up to one are conserved over time, everything you want a probability to be. In fact, in the paper that Chip Seamans and I, wrote to derive the born rule in many worlds, the way that I'd like to think about it, you know, we went through various hoops and math and so forth, but the moral of the story is the born rule. The assignment of the probability being the wave function squared is really just the perfectly obvious natural thing to do in quantum mechanics. Once you believe there are probabilities at all, those probabilities have to have certain properties, and those properties more or less uniquely point you toward the born rule.

2:37:56.0 SC: So the reason why David and Barry want to avoid that, well, I shouldn't, I'm not trying to psychoanalyze them, but they have to, to be consistent, deny the idea of self-locating probabilities, as a coherent thing. Because once you let that in, once you say, okay, you know, you have some uncertainty about which branch of the way function you're on, we have to assign some credences to them. What can we do? The answer is instantly the born rule and many worlds works, and you believe it, and that's a good position to be in. But if you're not gonna believe many worlds, then I think that denying the sensibility of having self-locating credences is a sensible thing to do. I just don't feel the urge to do that. Katie Spaghetti says, what are your thoughts on the philosophy and foundations of mathematics and the real numbers I.e the continuum do real numbers, infinite non-repeating decimals exist in the real world?

2:38:53.2 SC: You know, I don't know whether they do or not. I don't mind if they do. I think that it's undoubtedly the case that once a mathematical system becomes sufficiently powerful, it opens itself up to unfortunate [laughter] properties, right? Like Gödel's incompleteness theorems or various theorems about decidability and predictability and things like that. Various mismatches between sets of axioms and models of those axioms. The peano axioms, P-E-A-N-O, are famously axioms that, underlie arithmetic and the natural numbers, but not uniquely. They don't give you uniquely the integers that you know and love. There's other models of the peano, arithmetic axioms, and there's no way to just add an extra axiom that picks out the right one. So this leads people to mathematical Platonism and believing that we just know what the right model of the peano axioms is.

2:40:00.5 SC: And I, that none of that is at all, convincing to me. So therefore, it would be nice if we didn't have to use mathematical systems that were quite that powerful. I did recently write a paper on discrete, completely discrete quantum mechanics that pointed out that it is conceivable not easy and not even very nice, honestly, but conceivable that you could have a phenomenologically acceptable theory of quantum mechanics without the continuum, with only discreet numbers maybe. The big problem there is actually, believe it or not, ironically, Boltzmann brains, if you have a completely discreet quantum mechanics, you will generally have recurrences, and maybe not necessarily, but generally you will. I'm trying to think. If you have a completely discrete quantum mechanics in a finite dimensional Hilbert space, then you will generally have recurrences, and those include fluctuations and those include Boltzmann brains, which are bad, okay?

2:41:00.6 SC: So that's the big phenomenological problem that you have to get out of somehow, the cosmological issue. But otherwise, as far as the current universe is concerned, we can fit the data with completely discreet quantum mechanics, which is interesting to think about. I don't know whether that helps with the philosophy of mathematics and the various conundrums that arise there, but it's a, it's an open possibility. Brandon Lewis says, how do you feel about the new wave of fusion power startups? Do any of them seem promising? So I have no idea. I don't wanna answer the question in the sense of trying to provide an opinion about these things I have no idea about, but I thought I wanted to answer the question or address it because I wanna put in a little word in favor of fusion power. Nuclear fusion has been hyped for decades now, and the hype has often gotten out of control, and it's often turned out to be false, right?

2:41:58.1 SC: So I think that a certain weariness and cynicism sets in where people go, oh yeah, okay, another round of fusion hype. But look, David Hume would tell you, this kind of induction doesn't work. It is completely possible that a hundred years from now we will say, oh yeah, there's like 50 years of complete hype before we finally figured out how to do it, and it changed the world, right? That is an open possibility. I think it's okay to lower your credence in the usefulness of fusion based on the fact that many people have tried and failed, but it shouldn't be zero. So I'm very open to the idea in general that the technology has improved to such a place where soon we will have fusion power. I am not an expert. Don't, take that as the opinion of a physicist who has sat down and looked at the issues, because I have not, but I just wanna be open-minded to the idea that fields can be overhyped and nevertheless eventually pay off.

2:42:56.7 SC: Ryan Vaughn says, if the arrow of time arises from increasing entropy, why do we perceive time to advance at a constant rate one second per second when entropy increases at varying rates, say, inside a star versus interstellar space? Well, to be super duper clear, I often say this, but I'll say try to say it in slightly different words. The arrow of time is just the direction. It is not a magnitude, it is not a statement that there is something called the rate at which time goes. Entropy has nothing to do with one's perception of the rate at which time passes or anything like that. As you say, time passes at one second per second. The rate at which entropy is increasing has nothing to do with our perception of the passage of time. What it has to do with is we always perceive time to pass from the past to the future.

2:43:50.1 SC: That's all that it says. It is just a difference between past and future, not a quantitative rate of anything at all. Russ Dill says, in your paper with Bao and Singh on the Hilbert space of quantum gravity, you discuss the limited degrees of freedom of a local region of space and how it limits black holes. Does this also limit the compactness of the early universe, additionally, with the special low entropy state of the early universe allow it to be more compact than if it were in some other state? Well, no, not really. So the way to think about this, we actually talked about in a different paper called, quantum Circuit cosmology, also with Ning Bao, Charles Sao and Liam McAllister. And what we point out is if you buy the basic idea that space emerges from a bunch of entangled qubits and you want to explain how space expands, then what you do is you have a big bunch of qubits.

2:44:51.6 SC: Only some of them are entangled, the others are just not entangled and not participating in space itself, okay? So they exist in some sense, but they don't actually contribute to what you and I think of as space. So the nice thing about this way of thinking is you can understand in a quantum level what it means to say the universe is expanding. What it means to say that is these initially unentangled qubits are gradually entangling with the rest of the universe and making space bigger. Okay? So we wrote a little paper, pointing out that that is more or less, a sensible story. We don't have the full story. It's very, very far from a complete, well fleshed out theory, but there's no sort of logical, reason why it wouldn't work. And in fact, in my mind, it helps explain something that is otherwise very mysterious.

2:45:46.7 SC: When in the usual semi classical way of talking about the expansion of space, you have space expanding and you have quantum fields within that space, and these quantum fields, it is easiest to think about them in terms of waves whose wavelengths also expand along with the universe, which is fine and good. And if you do, inflationary cosmology or cosmological perturbation theory, you need to think in those terms. But it has the obvious problem that if you go backward in time, something like, the typical cosmic microwave background photon right now at what most cosmologists think of at as the beginning of the universe, or the beginning of inflation was much smaller than the planck scale. That's a problem. [laughter], you're not supposed to have things much smaller than the planck scale. What were those modes doing when they were smaller than the planck scale?

2:46:45.6 SC: So our picture answers that question, what were those modes doing? Namely, they were not entangled, they were not part of what you call space at all. It's not that they were trans plancking and somehow bubbled down to our world. They were always there, but they just weren't participating in the broader entangled group of qubits that were making up space as we know it. Okay, Pete Faulkner says, in the context of the cosmological multiverse, I often hear people say things like, if we wait long enough or somewhere very far away, when discussing phenomena such as Boltzmann brains or exact doppelgangers, if these phenomena are predicted due to purely random fluctuations, then surely the likelihood of them happening in the distant future or very far away are the same as them happening in the next five minutes or close by. They're both incredibly small. Yes, that is exactly true.

2:47:37.9 SC: So when we say if we wait long enough, we'll see a Boltzmann brain, we mean exactly the same thing as if you flip a coin enough times, eventually you'll see it come up heads a hundred times in a row. It's not that the rate or the likelihood is changing from moment to moment, it's just that you have to wait a very, very long time for the total probability to approach anything appreciable. The means of destruction says, how would you recommend a lay person conceptualize how time can become space and vice versa? I would not recommend the layperson conceptualize that at all. I don't even know exactly what it means for time to become space and relativity. Obviously, there's only one thing, namely space time, and there is a time-like direction and space like directions, and those are not completely uniquely fixed because it's relativity.

2:48:30.4 SC: And different observers will divide space time into time and space differently. But that's no different than saying, two people might lay down a grid to make a city differently, right? I mean, it's just a choice of coordinates. It's not that one is becoming the other as anything dynamical. So I don't know if that helps, but that's the best thing I can do for that question. Sid Huff asks a priority question. Imagine two spaceships flying in parallel, some distance apart, say 10 kilometers. There is a super strong long wire stretched between them. The ships fly on either side of a smallish black hole in space such that the wire cuts through the black hole. What would happen to the wire and to the black hole? I think you can work this out for yourself, honestly. Just be serious about what you're asking. If part of the wire falls into the black hole, that part is never coming out. That's all there is all she wrote. So the part that is just outside the black hole has two choices. Either it will be pulled into the black hole, in which case, the wires and the spaceships and everything just fall into the black hole or it breaks. Those are the only two options. I don't know exactly which option it will be. That depends on the wire, but one of those two things will happen.

2:49:51.4 SC: Qubit says, in your materialistic view of the world, even morality emerges from physical laws on the most fundamental level. Do you think there's any chance of finding a moral principle which attributes consciousness to some kind of exotic advanced computer which does not have any human-like properties? Certainly we do not feel compassion towards such a device, and therefore seems hard to imagine why we would attribute consciousness to it anyway. I have, no problem whatsoever in believing that a sufficiently advanced computer can have what you and I recognize as consciousness or even human rights or agency or anything like that. Again, I think that we're making a huge mistake of principle in how we typically talk and think about artificial intelligence in the modern world. We're just borrowing human words and applying them way too quickly to a very, very different thing. Human beings are embodied creatures.

2:50:49.2 SC: They're not designed by anybody. There's a whole bunch of things that go along with being human. Therefore, it is not at all obvious that anytime soon we will get anything that you and I recognize as conscious or deserving of compassion. Even if we do get things that fake it very, very well. And I say all that because I also want to say, nevertheless, I see no obstacle to eventually doing it. Human beings are just physical things, just like computers are. So no, if I'm consistent about it, I can't possibly say there is something about our squishy biology that makes us deserving of compassion, but not something that would be made of silicon. David Maxwell says, watching Oppenheimer, I found myself imagining what the world might be like. Had the US made theoretical physics an intergenerational priority. How much do you think the pace of advancement in theoretical physics is limited by the number of people doing it?

2:51:51.0 SC: Are there inherent limitations to the pace of change beyond the number of brain hours? I don't know. You know, the pace of progress in theoretical physics I think has been fine. [laughter], you know, you got to give nature some credit here. Okay? There's no rule that says we'll learn more about the universe if we just throw more brains at it. Probably there is some improvement in the rate of progress, but it's probably sublinear, right? It's not like if we have twice as many people doing theoretical physics, we'll make twice as much progress. I love theoretical physics and I think it's worthy of more support than it gets from, governments and things like that. But there's no way of knowing what the rate of progress is going to be, no matter how much money we throw at it. Maybe the progress would've been much greater because one person would've had a really good idea that we haven't had yet.

2:52:49.4 SC: Or maybe not. I don't know. So I think that the right thing to do is to, well, let's back up. If you are really a country or a nation or a society, you have a really, really hard problem in front of you, which is how to allocate your resources amidst a bunch of things that are worth doing, right? Theoretical physics is worth doing. So is experimental physics, so is biology, so is chemistry, so are history and literature and a whole bunch of other things, not to mention manufacturing and engineering and medicine and what have you. So nobody knows enough about the details of all of these things to really accurately do it. It's a very weird situation where certain members of those different sub communities have to make a case, right? They have to try to communicate to the rest of the world what is going on in their fields, why it is worth supporting, what can be done with the money and things like that.

2:53:49.7 SC: In my very, very tiny way, this is part of the goal of things like my books and my podcasts, just to let people know why these ideas are so interesting and important and then hopefully they decide that they're worth supporting. That's what I would like. But other things are worth supporting too. And I totally believe that. I sometimes wonder whether different academics aren't a little bit too honest about how, what they do is, well, it's just one of the things that is worth supporting. 'cause I mean, that's true, but if other areas of human endeavor are much more like just give me the money, then I worry that those areas are going to win. Okay, final question for this AMA comes from the great deceiver who actually, despite the, handle is going to ask quite a sweet question.

2:54:40.9 SC: A few years ago I was walking alone at night hitchhiking in Argentina, and I was treated to the most wonderful night sky that I've ever experienced and probably ever will experience. I could see clear across the galaxy. I'm wondering if you have ever been to South America to take in any of the amazing telescopes like the VLT in China or others, maybe in graduate school. What would you delight in peering at if you had full reign of one of those facilities? So let me get some of the [laughter], some of the annoying details out of the way first. Sadly, the VLT the very large telescope did not exist when I was in graduate school, so I couldn't look through it. And more importantly, there's no peering through it when it comes to these big telescopes. There's no IPs, right? There are photographic or other instruments, connected to them.

2:55:30.0 SC: Most of the time they're taking spectra, even not images. I think that various people have learned the PR value of taking images as well as the scientific value. So these days you get more images 'cause you have large CCD cameras, but it's the spectra which often reveal more information to the astronomers than the pictures do. Okay? Having said all that, I have been to South America several times and other parts of the Southern hemisphere and yeah, the sky is there. Beautiful. Also, the sky in the Northern hemisphere is also extremely beautiful if you're in a dark enough area. This is gonna sound weird, but as much as I love the night sky, I mean the Milky Way in particular gets to me. I don't need a powerful telescope because powerful telescopes would zoom in on some nebula or stars or planet or whatever.

2:56:21.3 SC: I prefer in my awe inspiring night sky experiences to look at the whole Milky Way all at once. Knowing that it's a big disc and that we are near the edge of it to me, like, how can you look at that and not get vertigo? Get a little dizzy knowing that where you are in the Milky Way. But having said that, looking at the night sky is not what gets me going, really what gets me going, and this is, as geeky as I'm ever gonna get, what gets me going is knowing that we can invent equations or discover equations depending on your philosophical preferences that correctly describe what's going on when we see those amazing things. And, and that's me. And I love astronomy and the night sky. I was an astronomy major. I had a part-time job showing people the night sky when I was an undergraduate.

2:57:13.7 SC: But, it's not what really gets me going, [laughter] It's the possibility of understanding the intellectual puzzle that we have been extraordinarily successful at making progress on. We're not nearly done, but what we've learned by doing science and thinking about what is happening out there in the sky and in the rest of reality is what really amazes me. And, keeps me excited to learn more and think about more. And I'm very glad that you all are here with me on the journey to learn more about our universe. So thank you very much. See you next week. Thanks for supporting Mindscape. Bye-bye.

[music]

4 thoughts on “AMA | October 2023”

  1. Who will challenge this basic monetary problem and/or the problem of circular logic and mathematical illiteracy when attempting ‘Applied Math’ in the field of economics? Maybe you? Genuine Scholars can change the course of history by challenging the status quo with its own scientific methods!!

    “Applied Math” is making mathematical calculations about things in the Real World.
    When using a term that is illogically defined and unspecified one can only be sure that the results of such calculations will be illogical and indeterminate.

    The term referred to in economic calculations is the dollar or Euro or other such monetary unit. The monetary unit is presently illogically defined and unspecified with reference to anything apart from itself. See Core Misrepresentation below.
    That is circular and unacceptable.

    So, starting from one’s own background in math, is it not the responsibility of the mathematician to alert the ‘authorities’ to this basic core error in logic as opposed to attempting to give credence to nonsense and direction that can only lead all further down this path of illiteracy?
    What is the responsibility for one trained in math AND outside the field of Economics? Most all schools in ‘higher education’ have a propensity to ignore and even defer to the illogical and illiterate use of Applied Math in the field of economics. This too is unacceptable.

    Academics must challenge the illiteracy of money if they wish to hold any stature within their own field. That or they will have to declare that their own field is subjugated to the illiteracy of present economic and monetary thinking.
    https://www.moneytransparency.com/core-misrepresentation

    It is Not Possible “to be in relationship to one another in an enhancing way” when using a system of money that determines that each must compete with the other just to do their acCounting!

    http://bibocurrency.com/index.php/downloads-2/19-english-root/learn/300-you-have-been-served

  2. Hi Sean listening to your ama this week,I was curious about the question with two space ships and a cable between them flying the cable through a black hole. You gave two options for the result, everything attached gets sucked in our the cable snaps, on a long enough time scale I’m guessing that’s true. But wouldn’t it also be possible that the ships would find themselves in a relatively stable orbit around the black hole, tethered by the cable?

  3. Interesting question and response about heroes. There are many definitions of a hero, and different opinions of what constitutes a heroic act. For those reasons, unless you know someone really well (e.g., a relative, loved one, close friend) it’s probably not reasonable to characterize them as a “good person”.

  4. As a former Physics Graduate Student who switched careers, I always felt my education was deficient without GR.

    We had a focus on quantum mechanics but not GR. I felt reassured with Sean’s comments about the lack of GR amongst the general physics population.

    30 years later, I have now watched all of the Suskind Theoretical Minimum GR lectures, and others, and have his book. I also have Sean’s GR textbook and am (slowly) working on that!

Comments are closed.

Scroll to Top